SIE Q's

¡Supera tus tareas y exámenes ahora con Quizwiz!

A customer buys a 4% Treasury bond, maturing in 10 years, at a price of $96.08. The yield to maturity (YTM) is A) same as current yield. B) less than current yield. C) greater than nominal yield. D) less than nominal yield.

A bond whose price is below par (priced at a discount) has a higher YTM than current yield, which in turn is higher than the nominal yield. LO 3.a C

A quote for Seabird Airlines (SBRD) is 17 B 17 ½ A 5 x 5. A customer would be able to sell how many shares and at what price? A) 5 shares at $17 a share B) 500 shares at $17.50 a share C) 1,750 shares at $5 a share D) 500 shares at $17 a share

A quote is the bid and ask prices followed by the size on each side. A customer sells at the bid ($17) and the size is in round lots (5 × 100 = 500 shares). LO 1.f D

Which of the following issues only common stock? A) Corporations B) Closed-end funds C) Municipalities D) Mutual funds

An open-end investment company (like a mutual fund) only issues one class of security, which is common stock (no preferred shares or bonds). Closed-end funds may issue common shares, preferred shares, and bonds. Municipalities do not issue stock. Corporations may issue common stock, preferred stock, and debt. LO 4.a D

Which of these investment companies trade in the secondary market? A) Face amount certificates B) Unit investment trusts C) Open end funds D) Closed end funds

Another name for closed end funds is publicly traded funds because they trade in the market like stock of other companies. The other three investment companies listed here are purchased and redeemed through the issuer (a primary market transaction). LO 4.a D

To the benefit of the bondholder, a puttable bond is likely to be put back to the issuer when interest rates A) fall. B) are volatile, moving both up and down over short periods of time. C) remain stable for long periods of time. D) rise.

Bonds with put features are most likely to be put back to the issuer when interest rates rise. For example, if a bondholder has a bond paying 4% and interest rates have risen to 6%, why settle for a 4% return when prevailing market rates are now up to 6%? Better to put the 4% bond back to the issuer for redemption and then purchase a new bond paying the prevailing higher rate. In this way, put features benefit the bondholder. LO 3.a D

In a reverse split, what happens to the cost basis and number of shares? I. The cost basis goes up II. The number of shares goes down III. The cost basis goes down IV. The number of shares goes up

For reverse splits, the price goes up and the number of shares goes down. LO 2.h B

A Federal Reserve member bank's deposits in excess of the amount required to be on reserve are known as A) jumbo funds. B) promissory funds. C) prime funds. D) federal funds.

The Federal Reserve Bank (FRB) mandates how much money its member banks must keep on reserve at the Federal Reserve. Any deposits in excess of the required amount are known as federal funds. LO 3.f D

In order to accelerate growth in a slowing economy, demand-side theory would suggest which of the following steps? A) Raise taxes B) Lower taxes C) Decrease regulation D) Decrease spending

Basic demand-side (Keynesian theory) would stimulate the economy by lowering taxes and increasing government spending. LO 9.e B

Stock ledgers, a record of stocks owned by the firm, are maintained for at least A) six years. B) four years. C) three years. D) ten years.

Financial and holdings records are normally retained for six years. LO 10.g A

LMN Corporation has a $60 par, 4% preferred stock currently trading at $45 per share. Its annual dividend is A) $24.00. B) $4.00. C) $1.80. D) $2.40.

For preferred shares, the annual dividend is stated as a percentage of par. In this case, 4% of par value of $60 equals $2.40. LO 2.d D

The aftermarket prospectus requirement for the IPO of nonlisted securities is A) 90 days. B) 40 days. C) 25 days. D) not specified in the Securities Act of 1933.

For the first 90 days following the IPO, a prospectus must be provided to purchasers in the secondary market. LO 1.b A

The risk that an investor might not be able to sell an investment quickly and at a fair market price is known as A) financial or default risk. B) inflation or purchasing power risk. C) call or reinvestment risk. D) liquidity or marketability risk.

Having investments that are liquid means being able to divest of them quickly at a fair price. Liquidity risk comes for investors holding assets where doing that might not be possible. LO 7.a D

The rules to prevent pay to play regarding contributions made to political parties, candidates, and elected officials by firms involved in the underwriting or sales of municipal securities are enforced by A) Financial Industry Regulatory Authority (FINRA). B) Securities and Exchange Commission (SEC). C) Federal Reserve Board (FRB). D) Municipal Securities Rule Board (MSRB).

Having no authority to enforce the rules it enacts, the MSRB relies on FINRA to enforce its municipal securities rules. This would include the enforcement of the pay to play or play for pay rules regarding political contributions. LO 12.f A

Alan and Barbara Collins have three minor children: Dan, Ellen, and Frank. Which of the following UTMA accounts could be opened? A) Alan Collins as custodian for Dan and Frank Collins B) Barbara Collins as custodian for Ellen Collins C) Frank Collins as custodian for Dan Collins D) Alan and Barbara Collins as custodians for Ellen Collins

In an UTMA account, one adult is custodian for one minor. There is no such thing as joint custodians or joint beneficiaries. LO 6.d B

Regarding the decision to dissolve a LP before its scheduled predetermined dissolution date, it would need to be A) voted on by the general partner(s) only. B) voted on by the limited partners holding a majority interest. C) made by the general partner with the largest capital contribution with no vote required. D) ratified by the IRS because of the tax implications to dissolve earlier than planned.

In instances where a decision to dissolve a limited partnership before its predetermined date is made, an affirmative vote to do so must be taken by the limited partners. LO 5.f B

Your customer establishes the following position: Long 1 XYZ January 50 put at 2. You can correctly inform the customer that the maximum potential gain on the position is A) $5,200. B) $200. C) unlimited. D) $4,800.

Maximum gain for a long put is calculated by subtracting the premium from the strike price (50 − 2 = 48 per share). One contract represents 100 shares, so the buyer's maximum gain is $4,800 (this occurs if the stock becomes worthless). LO 5.a D

All of the following statements about a bond selling above par value are true except A) the yield to maturity is lower than the nominal yield. B) the nominal yield is lower than the yield to maturity. C) the nominal yield always stays the same. D) the current yield is higher than the yield to maturity.

Nominal (coupon) yield is fixed and stays the same with all bonds. A bond selling above par is selling at a premium, so the yield to maturity is lower than the current yield—which, in turn, is lower than the nominal yield. LO 3.a B

All of the following statements about a bond selling above par value are true except A) the nominal yield always stays the same. B) the current yield is higher than the yield to maturity. C) the yield to maturity is lower than the nominal yield. D) the nominal yield is lower than the current yield.

Nominal (i.e., coupon) yield is fixed and stays the same with all bonds. A bond selling above par is selling at a premium, so the yield to maturity is lower than the current yield—which, in turn, is lower than the nominal yield. LO 3.a D

Considering a customer's nonfinancial considerations is as important as considering the customer's financial concerns. Included in the category of nonfinancial considerations are I. salary. II. marital status. III. credit card debt. IV. number and ages of dependents. A) II and IV B) II and III C) I and III D) I and IV

Nonfinancial considerations are those that are not generally monetized. Even so, they can sometimes be more important than the financial ones. Things like marital status and the number and ages of the customer's dependents play a critical role in determining the appropriate investment strategies. Of course, knowing the salary and debt is important, but those are financial considerations, not the subject of this question. LO 7.d A

A strategy that blends various types of investment in an effort to reduce nonsystematic risk is called A) capital preservation. B) dollar cost averaging. C) diversification. D) sector rotation.

Nonsystematic risk occurs based on circumstances or events that are unique to a specific security. This risk can be managed by diversifying the assets in a portfolio by selecting securities that possess different risk and/or return characteristics. LO 7.b C

Which of the following statements regarding systematic risk as it relates to an investment portfolio is true? A) Diversification cannot mitigate it to any extent. B) Diversification can be used to eliminate it completely. C) Diversification will not eliminate it. D) Diversification ensures that portfolios are not subject to it.

Systematic risk is the risk that changes in the overall economy will have an adverse effect on individual securities, regardless of the company's circumstances. Understanding what it is, is to know that no amount of diversification will eliminate it completely. While one might be able to mitigate it somewhat, one cannot diversify away systematic risk. LO 7.b C

T-bonds are the U.S. government's A) short-term debt of 1 year or less. B) intermediate-term debt of 2-10 years. C) long-term debt of over 10 years. D) only tax-free debt.

T-bonds have a maturity in excess of 10 years when issued. LO 3.e C

As a part of its anti-money laundering (AML) policies, every financial institution must appoint A) a chief compliance officer. B) a bank security associate. C) an AML officer. D) a privacy officer.

The Bank Secrecy Act requires that all financial institutions appoint and empower an officer to create and implement AML procedures. LO 11.b C

How long do securities regulators retain jurisdiction over a person after a U5 form is filed? A) 180 days B) 10 years C) 5 years D) 2 years

Regulators retain jurisdiction for a period of two years from termination. A regulator may investigate and hold hearings for actions taken while employed during this period. LO 12.a D

Which of the following securities are exempt from the Securities Act of 1933? A) Treasury bond funds B) Treasury notes C) 30-year AAA corporate bonds D) Common stock

Securities issued by the federal government are exempt from the Securities Act of 1933. A mutual fund that invests in these securities is not. LO 1.c B

A form of market manipulation that attempts to keep the price of the stock from falling is called A) front running. B) backing away. C) supporting. D) capping.

Supporting is a form of market manipulation to keep the price of a security from falling. It is illegal if used to manipulate the market. LO 11.c C

If a broker-dealer suspects that a transaction involves funds derived from illegal activity, a suspicious activity report (SAR) would be triggered at what threshold? A) At least $5,000 in funds or other assets B) More than $5,000 in funds or other assets C) More than $10,000 in funds or other assets D) At least $10,000 in funds or other assets

The threshold for triggering a suspicious activity report (SAR) is at least $5,000 in funds or other assets. Do not confuse this with a Currency Transaction Report (CTR), which is triggered by amounts greater than $10,000. LO 11.b A

DEF Growth Fund has total assets of $330 million and liabilities of $15 million. The fund has 10 million outstanding shares. What is the fund's current net asset value (NAV) per share? A) $32.00 B) $33.00 C) $34.50 D) $31.50

To find the net assets, you subtract the fund's liabilities from the fund's total assets. In this question 330 million - 15 million = 315 million. NAV is calculated by dividing the net assets of the fund by the number of outstanding shares. In this question 315 million / 10 million = $31.5 per share. LO 4.c D

When the Federal Open Market Committee (FOMC) directs that Treasury securities be purchased in the open market, this A) decreases the money supply. B) is intended to hinder contraction of the money supply. C) increases the supply of money. D) stabilizes the money supply.

When the FOMC directs that Treasury securities be purchased in the open market, this increases the supply of money. Treasury securities are coming out of the economy and, therefore, money is going in—the money supply increases. LO 9.h C

A broker-dealer's customer will be relocating for a position with a higher salary and bonus potential. This requires A) reviewing investment objectives within the next 90 days. B) opening an account with a broker-dealer in the new location. C) notification to the broker-dealer of the change within 30 days. D) completing a new account application within 30 days.

When there are significant changes to the client's status, such as salary and bonus potential as well as change of address, the client should notify the member firm within 30 days. LO 11.k C

Which of the following is not a feature shared by mutual funds and closed-end funds? A) A professional investment adviser manages the portfolio for investors. B) The fund may offer various withdrawal plans that allow different payment methods at redemption. C) An investor may liquidate a portion of his holdings without disturbing the portfolio's balance or diversification. D) The fund provides diversification by investing in different companies or securities.

A closed-end fund does not offer redemptions. Shares must be liquidated in the secondary markets. All the other points here are true of both types of managed investment companies. LO 4.b B

All of the following are elements of a customer's suitability except A) their alma mater. B) their current profession. C) their liquid net worth. D) their objectives.

A customer's school is not an element of suitability, though it is a good thing to know. The other responses are mandatory elements of suitability. LO 7.d A

Overprice.com, Inc. (ticker: OVER) is an NYSE-listed big box retailer. The company has a long history of steady growth and has paid a dividend every quarter for five decades. It recently announced that it expects steady growth in same-store sales and will continue paying dividends into the foreseeable future. It has also announced plans to expand into Central and South America. OVER stock is an example of A) a preferred stock. B) a blue-chip stock. C) a mid-cap stock. D) an emerging market stock.

Blue-chip stocks are the equity issues of financially stable, well-established companies that have demonstrated their ability to pay dividends in both good and bad times. LO 2.a B

A bond's rating is used primarily as a measure of its A) interest rate risk. B) volatility risk. C) purchasing power risk. D) default risk.

Bond ratings from credit rating agencies are used to compare the relative risk of default. None of the others are issues of default. LO 3.b D

Your customer is a resident of the state of Utah. She owns bonds issued by Puerto Rico. The interest from these bonds is A) taxable at the state and local level because she is not a resident of Puerto Rico, but still tax free at the Federal level. B) taxable at the state level only. C) tax free at all levels for U.S. citizens. D) taxable at all levels because the bonds are not issued by a state.

Bonds issued by or from a territory of the United States have tax-free income at all levels to U.S. citizens. LO 3.g C

All of the following statements about securities purchases are true except A) in a short margin account, customers borrow securities for short sales. B) in a long margin account, customers borrow money for securities purchases. C) in a cash account, the customer pays in full for securities. D) securities may not be purchased with borrowed money.

Borrowing is a perfectly acceptable practice when buying and selling securities, whether it is cash that is borrowed for purchases or securities that are borrowed, chiefly from other investors who have signed a loan consent agreement, allowing their securities to be borrowed for short sales. LO 6.g D

Borrowing money to buy securities is prohibited in all of the following accounts except A) a custodial account. B) a margin account. C) a Roth IRA. D) an individual IRA.

Borrowing money to buy securities can only be done in a margin account. Retirement accounts and custodial accounts do not allow margin. LO 6.g B

What rights do owners of American depositary receipts (ADRs) and owners of domestic common stock share? A) The right to vote the shares B) The right to transfer the asset freely C) The right to maintain percentage of ownership D) The right to convert to the underlying asset

Both are freely transferable assets (as securities that trade freely in the secondary markets). Holders of ADRs have the right to request that they receive the underlying stock in place of the ADR. Depositary banks are not required to pass voting or stock rights through to the holder of the ADR. These last two points are not in the LEM, but the correct answer may be arrived at by process of elimination. LO 2.e B

ABC Broker-Dealer maintains an inventory of MMNO common stock. They transact purchases and sales with customers using this inventory. In this example, ABC is acting as what? A) An agent B) A broker C) A market maker D) A transfer agent

Broker-dealers who incorporate proprietary trading into their business model are known as market makers. As a market maker, the broker-dealer trades in their own account attempting to profit. A firm making markets may be a carrying firm or a fully disclosed firm. Commissionable transactions are those done by brokers for customer accounts not proprietary trades. LO 1.h C

A company is about to introduce a new product. While confident in the product's appeal and market, it is still an unknown factor until sales results are viewed later. Investors holding stock in the company are at this time specifically exposed to A) call risk. B) business risk. C) financial risk. D) reinvestment risk.

Business risk is an operating risk related to poor or untimely management decisions. Decisions regarding if and when to introduce new products are one example of those that might expose investors specifically to business risk. LO 7.a B

Which of these risks are not normally associated with bonds? A) Default risk B) Purchasing power risk C) Interest rate risk D) Business risk

Business risk is related to the growth prospects of a business and is most closely associated with common stock. Bond prices are subject to changes in interest rates. Default occurs when a company fails to meet its obligations to the bond holders. Most bonds are subject to a loss of purchasing power due to inflation. LO 3.b D

When a broker-dealer pledges customer securities to a bank as collateral for a margin loan, the pledge is known as A) rehypothecation. B) loan consent. C) credit agreement. D) hypothecation.

By signing the margin agreement, a customer hypothecates (pledges) the securities to the broker-dealer who then rehypothecates (pledges) them to the bank as collateral for the margin loan. LO 6.g A

Which of the following business structures allows the pass through of the business results and liability protection? A) Partnership B) C corporation C) Sole proprietorship D) Limited liability company

C corporations do not pass through the business results to the shareholders. Sole proprietorships and partnerships provide no liability protection. Of the options available, only an LLC does both. LO 6.b D

A call or put that can be exercised before expiration is A) an Eastern-style options. B) a European-style options. C) an American-style option. D) a Western-style options.

Call or put buyers can exercise a contract any time before expiration if the contract is an American-style option. European-style options can be exercised on expiration day (last day of trading) only. Nearly all equity options are American style. Foreign currency options may be either American-style or European-style. LO 5.b C

All of the following are exempt from the do not call provisions of the Telephone Consumer Protection Act except A) calls made by tax exempt nonprofit organizations. B) calls made for legitimate debt collection. C) calls made to make the customer aware of new products that are available to the customer. D) calls not made for commercial purposes.

Calls made to make the customer aware of new products that are available to the customer are solicitation calls and are what this act is all about. The others are specifically exempted from the act. LO 11.j C

The statement "These securities have not been approved or disapproved nor have any representations been made about the accuracy or the adequacy of the information" is A) placed by the issuer in the preliminary prospectus. B) mandated to be in the final prospectus by the Securities and Exchange Commission (SEC). C) mandated by the Financial Industry Regulatory Authority (FINRA) to be placed in both the preliminary and final prospectus. D) is the disclaimer placed by the underwriters in a tombstone advertisement.

Commonly known as the Securities and Exchange Commission's disclaimer, the SEC mandates that it be found in the final prospectus. LO 1.b B

All of the following are true of Roth IRAs except A) Contributions may be able to be made after 59½ B) Withdrawals are not required at age 72 C) Contributions may be deductible depending on income limits D) Contributions are made after tax

Contributions are not deductible. They are made with after-tax dollars and may continue past age 59½ if still working. Roths are not subject to RMDs. LO 6.e C

Which of these would not be included in a mutual fund's list of expenses? I. Shareholder records and service II. Investment adviser's fee III. Broker-dealer sales charges IV. Underwriter's sales loads A) III and IV B) I and III C) I and II D) II and IV

Costs to maintain shareholder records, costs to provide services to shareholders, and the investment adviser's fees are all expenses to the fund. The costs paid in the form of sales charges (loads) to an underwriter, or broker-dealers selling mutual funds to the public may never be treated as an expense to the fund. These are expenses to the investor. LO 4.c A

Individual retirement accounts allow a catch-up contribution of $1,000 to be made into the account for those who are A) 59½ years old or over. B) over 72 years old. C) 50 years old or over. D) over 50 years old.

Catch-up contributions are for those ages 50 and over (not over 50). LO 6.e C

Class B mutual fund shares are also called A) CDSC shares. B) reverse load shares. C) back-end load shares. D) deferred-load shares.

Class B mutual fund shares are bought with no sales charge at the time of purchase. The sales charge is paid instead at the time of redemption, or at the back end. Hence, they are known as back-end load shares. For this type of share, the sales charge percentage is reduced each year of ownership, typically becoming zero after five years. At this time, they convert to Class A shares. LO 4.b C

Class C mutual fund shares are also known as A) intermediate shares. B) periodic-load shares. C) successive-load shares. D) level-load shares.

Class C mutual fund shares have no sales charge at the time of purchase, but have a percentage of their value withdrawn from the customer's account every quarter. The percentage, which can be as high as 0.75% of the value of the account, never ceases. The charge remains as long as the account does—it remains level—hence, the shares are known as level-load shares. LO 4.b D

Which class of shares have a 12b-1 fee as the primary sales charge? A) Class C shares B) Class A shares C) Class B shares D) No load

Class C shares charge a level load built into the expense ratio, usually as a 12b-1 fee. Class B shares have back-end loads that reduce over time (contingent deferred sales charge, or CDSC). Class A shares charge an upfront load. No load funds have no sales charge. LO 4.b A

Which of the following may be purchased on a stock exchange? A) Common shares of an open-end investment company B) Face-amount certificates C) Common shares of a closed-end investment company D) Units of a unit investment trust

Closed-end investment companies are often called publicly traded funds. After the stock is sold in the initial offering, anyone can buy or sell shares in the secondary market (i.e., on an exchange or over the counter) in transactions between private investors. LO 4.a C

All of the following are exempt issuers except A) the City of Alta Loma. B) Alta Loma Community Foundation. C) the Southwest Railroad Co. D) Modulux, Inc., a home manufacturer.

Common carriers (e.g., railroads), municipalities, and charities are all examples of exempt issuers under the Securities Act of 1933. A for-profit corporation is not exempt. LO 1.c D

The risk of being the last to get paid in a corporate liquidation is characteristic of which of the following? A) Preferred stock B) Subordinate debentures C) Common stock D) Secured debt

Common stock is always last to get paid. LO 3.c C

All of the following are securities that are exempt from registration except A) BigCity National Bank Holding Company, Inc., 180-day commercial paper. B) a five-year Treasury note. C) BigCity National Bank Holding Company, Inc., common stock. D) ABC Corporation 90-day commercial paper.

Common stock of a corporation is not exempt from registration. Securities with maturities of 270 days or less are exempt. The Treasury is an exempt issuer. LO 1.c C

Which of these would cause a change in the net asset value of a mutual fund share? A) The market value of the portfolio declines B) The fund takes a new position C) Many shares are redeemed D) Securities in the portfolio are sold for a capital gain

A decline in the market value of the portfolio would reduce the assets of the fund without changing the number of outstanding shares. Sales and redemptions of shares change the net assets but also change the number of shares outstanding to the same degree, leaving the NAV per share unchanged. Buying or selling securities for a capital gain simply replaces securities in the portfolio, with an equivalent amount of cash, leaving the NAV unchanged. LO 4.c A

A fiduciary would be best described as A) a guardian designated by the courts to act on behalf of those investors who are unable or unwilling to satisfy a margin call. B) a voting board member of the Federal Deposit Insurance Corporation (FDIC). C) a beneficial owner of an individual cash or margin account. D) any person legally appointed and authorized to represent another person, act on that person's behalf, and make whatever decisions are necessary to the prudent management of the account.

A fiduciary is a person legally appointed to represent another person, and make whatever decisions are necessary to prudently manage owner's account. The investments exist for the owner's beneficial interest, yet the owner has little or no legal control over them. The fiduciary makes all of the investment, management, and distribution decisions and must manage the account in the owner's best interests. LO 6.d D

ACE, an open-end investment company, operates under the conduit, or pipeline, tax theory. Last year, it distributed 91% of all net investment income as a dividend to shareholders. Therefore, which of the following statements is true? A) ACE paid no taxes last year because it qualified as a regulated investment company under IRC Subchapter M. B) ACE paid taxes on 91% of its net investment income last year. C) ACE paid taxes on 9% of its net investment income last year. D) ACE paid taxes on 9% of its net investment income and capital gains last year.

ACE pays taxes on any portion of income it does not distribute, as long as it distributes at least 90%; ACE paid taxes on 9%. LO 4.e C

A broker-dealer firm's registration to do business in a given state may be revoked by A) the SEC. B) FINRA. C) the state's administrator. D) NASAA.

Actions taken against a broker-dealer in a specific state will be taken by that state's securities administrator. The Federal Reserve and the SEC are national level regulators and any actions taken will not be state specific. NASAA is an association of state securities administrators. NASAA writes model rules, but has no regulatory authority. LO 10.d C

During times when interest rates are rising, which of the following preferred are likely to pay a higher annual dividend? A) Convertible B) Participating C) Adjustable rate D) Callable

Adjustable-rate preferred dividends are tied to benchmark interest rates such as Treasury securities. As these rates fluctuate up and down, so do the dividends on the adjustable shares. LO 2.d C

Which of the following is part of the expense ratio of a mutual fund? A) Sales loads B) Deferred sales charges C) Administration fees D) Contingent sales charges

Administrative costs are part of the expenses of a mutual fund and by extension are part of the expense ratio. The expense ratio does not include sales charges or loads. LO 4.c C

T-notes are delivered in A) physical certificates. B) registered as to principal only form. C) book entry. D) bearer form.

All U.S. government issues are delivered in book entry. LO 3.e C

Which of the following must precede the first trade in an account? A) Customer's notarized signature on the new account form B) Filing of the account information with the applicable self-regulatory organization (SRO) C) Registered representative's signature on the new account form D) Approval of the new account by a principal

All accounts must be approved by a principal before the first trade. Neither the customer nor the registered representative need sign the new account form, and no self-regulatory organization (SRO)—requires the filing of new account information with them. LO 6.g D

Industry rules regarding political contributions intended to preserve investor confidence and market integrity apply to contributions made to A) elected officials and political candidates only. B) political parties, candidates for office, and elected officials. C) political parties only. D) political parties and third parties with connections to them only.

All business should be awarded on the basis of merit only and not political favor gotten via contributions to political parties, elected individuals or candidates, or third parties with connections to those with political affiliations. LO 12.f B

Regulation S-P requires that financial institutions provide information to customers and consumers concerning A) their policies on the protection of nonpublic personal information. B) the categories of nonpublic personal information that a broker-dealer, fund, or registered investment adviser may collect and disclose. C) a consumer's right to opt out of the disclosure of nonpublic personal information to nonaffiliated third parties. D) all of these.

All of these are part of Regulation S-P. LO 11.k D

Your customer is a limited partner in a real estate partnership. This partner has the right to do all of the following except A) inspect and obtain copies of all partnership records. B) sue the general partner for damages resulting from any business decisions made. C) vote with the limited partners to remove the general partner. D) choose which properties the partnership should buy or sell.

All of these are rights of the LP, except choosing the assets to be purchased for the partnership. This is a function of the general partner (GP). LO 5.f D

On Tuesday, July 3, your customer bought 15 SBRD 30 September calls at 4. On Friday, August 10, the calls are in the money and your customer issues exercise instructions. On which days did the trade and the exercise settle? A) July 6 and August 13 B) July 4 and August 11 C) July 5 and August 12 D) July 5 and August 14

All option trades settle next business day. The exercise of an equity option settles in two business days. July 4 is a holiday. August 11 and 12 are on a weekend. July 4 and December 25 are the only holidays we expect you to see on the test. LO 1.i D

When XYZ is trading at 40, an XYZ 30 put sold at 3 would be A) at parity. B) in the money. C) at the money. D) out of the money.

All puts are in the money when the market price is below the strike price. They are out of the money when the market price is above the strike price. They at the money when the market price equals the strike price. They are at parity when the premium equals the intrinsic value. LO 5.a D

A customer has given permission for securities in an investment account to be used for the purpose of other customers who want to borrow them in order to sell those securities short. This would have entailed the customer signing A) a margin account form. B) a credit agreement. C) a hypothecation agreement. D) a loan consent form.

Allowing one's securities to be loaned to others who want to borrow them for the purpose of selling them short would entail signing a loan consent form. This is optional and need not be signed to open a margin account or any other. LO 6.g D

Which of these is in correct order of priority for a corporate liquidation? A) Convertible bonds, participating preferred stock, common stock, subordinated debentures B) Guaranteed bond, secured bond, debenture, common stock C) Subordinated debenture, participating preferred stock, common stock, convertible preferred stock D) Secured bond, debenture, subordinated debenture, common stock

Any debt issue is superior to any equity issue. Any preferred stock is senior to any common stock. A guaranteed bond is unsecured debt, or a debenture. LO 3.c D

How long must record of a customer complaint be retained? A) Indefinitely B) Four years after receipt C) Four years after resolution D) Six years after resolution

Any record of written complaints must be kept for four years after resolution. LO 10.g C

Which of these would be considered a recommendation? I. You suggest that the client consider adding the Windmill Growth Fund to their portfolio. II. You explain to a customer how a sell stop limit order works. III. When speaking to a client you notice they are carrying a large cash balance. You suggest they move those funds into the firm's tax-free money market fund. IV. You enter an order for a trade. A) I and III B) II and III C) II and IV D) I and II

Any suggestion to a client to invest funds in a security is a recommendation, including the suggestion to move a free credit into a money market fund. Explaining how a trade limit works is not a recommendation. A trade may be the result of a recommendation, but the trade, itself, is not. LO 7.c A

Which of the following would increase the U.S. balance of payments deficit? A) An increase in exports of domestic goods from the U.S. B) A decrease in purchases of U.S. securities by foreign investors. C) A decrease in imports of foreign goods into the U.S. D) A decrease in dividend payments by U.S. companies to foreign investors.

Anything that will bring foreign money to the U.S. will decrease the balance of payments. Foreign investors pulling their money out of the U.S. or investing less in the U.S. will increase the U.S. deficit. LO 9.f B

Bob's Discount Brokerage applied for FINRA membership and was denied. Bob wants to appeal this decision. The first appeal goes to A) the courts. B) the SEC. C) the DOE. D) the NAC.

Appeals on FINRA decisions first go to an internal group called the National Adjudicatory Council. NAC decisions may be appealed up to the SEC, and then the courts. LO 10.e D

The relationship between fixed-income prices and prevailing interest rates is A) adverse. B) inverse. C) reverse. D) coterminous.

As interest rates (yields) increase, the price of outstanding debt decreases and vice versa. The resulting relationship is called an inverse relationship. LO 3.a B

Which of these Treasury securities is in correct order of shortest to longest maturities? A) Notes, bonds, bills B) Notes, bills, bonds C) Bills, notes, bonds D) Bonds, notes, bills

Bills have the shortest maturities with a maximum of one year (52 weeks), notes are from two to ten years, and bonds have maturities of more than ten years. LO 3.e C

An investor has a cash account with $300,000 in securities and $40,000 in cash. The investor also has a restricted long margin account containing securities with a market value of $220,000 and equity of $60,000. What is the extent of this investor's Securities Investor Protection Corporation (SIPC) coverage? A) $100,000 B) $280,000 C) $400,000 D) $620,000

Coverage under SIPC may not exceed $500,000 in cash and securities, of which up to $250,000 may be cash. In the cash account, his coverage is $300,000 in securities plus $40,000 in cash. In the long margin account, the coverage is only the equity, which is $60,000. Total: $300,000 + $40,000 + $60,000 = $400,000. LO 10.c C

All of the following are benefits of Coverdell Education Saving Accounts except A) they are available for use for K-12. B) no income restrictions. C) they can be transferred to a sibling if not used by the original beneficiary. D) withdrawals are tax free if used for qualified education expenses.

Coverdell plans have income restrictions; Section 529 plans do not. All the other benefits listed here apply to Coverdell plans. LO 6.c B

Records relating to a Currency Transaction Report (CTR) must be retained for A) three years. B) four years. C) six years. D) five years.

Currency Transaction Reports (CTRs) must be retained on file, together with other records generated in conjunction with them, for five years. LO 11.b D

Written customer complaints must be kept for how many years? A) Two years by the member firms B) Six years by the SEC C) Five years by the SEC D) Four years by the broker-dealer

Customer complaints is the only record that must be kept for four years. LO 10.g D

Seacoast Securities is a market maker for JIM common stock. They complete a sale of 300 shares of JIM common to a customer from inventory and charge a modest commission for the transaction. This activity is an example of which of the following? A) Acting as a principal agent B) Acting as a dealer C) Acting as a broker D) Acting in both a broker and dealer capacity on the same trade

D

For a new issue that qualifies for listing on an exchange, a prospectus must be provided to all purchasers for how many days after the effective date? A) 40 days B) 60 days C) 90 days D) 25 days

For new issues that qualify for listing on an exchange or Nasdaq (NMS securities), the prospectus delivery requirement period in the aftermarket (after the effective date) is 25 days. For nonlisted and non-Nasdaq (non-NMS) securities, the period is 90 days. For an APO, the reqquirement for NMS securities is 0 days (no requirement). The APO of a non-NMS security is 40 days. LO 1.b D

The Alta Loma High School District is asking voters to approve a bond to fund the purchase of new computers and software. The bond will mature in 40 years and the interest and principal payments will be funded from real estate taxes. This is an example of a A) GO bond. B) revenue bond. C) a debenture. D) an equipment trust bond.

If a municipal bond requires a vote it is most likely a GO bond. Generally revenue bonds do not require a vote (note that there is no revenue generating source here). Debentures and equipment trust certificates are issued by corporations, not municipalities LO 3.d A

The economy is showing that employment is low, there is little consumer demand, and loans for expansion and retooling are way down, showing a lack of business activity. Yet prices for consumer goods are still rising. Economists would call this a period of A) stagflation. B) stagnation. C) inflation. D) deflation.

Inflation is characterized by a rise in prices for goods and services. Stagnation is characterized by high unemployment and lack of growth and business activity. When these occur simultaneously, economists refer to these times as periods of stagflation. LO 9.b A

Of the following, which would not be considered institutional communications with the public? A) An internal memo promoting a new product that will be offered to your firm's institutional customers only B) A letter to a municipality offering your firm's services as an underwriter C) A communication with an individual designated to act on behalf of your institutional customer D) A letter to another broker-dealer regarding potential business together

Institutional communications specifically exclude internal communications such as memos. Communications with another member firm, a government entity, such as a municipality or with someone designated to act on behalf of one of your firm's institutional customers, would all fall within the definition of institutional communications. LO 11.i A

Institutional trading desks that place trades away from the visible markets in order to avoid impacting public quotes and revealing trading strategies are using which of the following? A) Third market B) Dark pools of liquidity C) Exchanges D) Over-the-counter market

Institutional trading desks that choose to use dark pools are able to execute large block orders without impacting public quotes or price or revealing their investment strategy for any of their holdings. Additionally, orders can be placed anonymously so that the identity of the entity placing the order—along with the volume and price for the transaction—is unknown to the general investing public. LO 1.d B

When a customer chooses to annuitize a variable annuity, all of these are factors the insurance company will use in calculating the initial payout amount except A) gender of the annuitant. B) age of the annuitant. C) historic inflation rate. D) balance of the separate account.

Insurance companies do not consider inflation when making this calculation. The components are GAAPI: gender, age, account balance, payout option, and interest rate (AIR). LO 4.f C

Interest-rate risk A) is often called purchasing power risk. B) occurs when interest rates fall, pushing bond prices lower. C) cannot be reduced by diversification. D) occurs when interest rates rise, pushing bond prices higher.

Interest-rate risk is one of the systematic risks that cannot be reduced by diversification. It is the risk that fluctuating interest rates will impact bond prices. Primarily, when interest rates are rising, bond prices will be pushed lower. LO 7.a C

What is the intrinsic value of an XYZ 40 call bought at a premium of 3 when the current market value of XYZ is at 30? A) -$10 B) $7 C) -$7 D) $0

Intrinsic value is the amount that a contract is in the money. The premium of the contract is not a factor. All calls are in the money when the market value of the stock is above the strike price. LO 5.a D

Where on a balance sheet would you find the estimated value of a retail company's inventory? A) Current liabilities B) Current assets C) Net worth D) Fixed assets

Inventory is reported in the current assets of a company's balance sheet. LO 9.d B

Under the Investment Company Act of 1940, which of the following is not considered an investment company? A) Unit investment trust B) Face-amount certificate company C) Hedge fund D) Separate account within a variable annuity

Investment companies include face-amount certificates, unit investment trusts, and management companies (both open- and closed-end). The separate account within a VA is a type of open-end management company. Hedge funds are organized as private investment companies (often limited partnerships), which are excluded under the definition of investment company under the Investment Company Act of 1940. LO 4.a C

Regarding the taxation of gains on securities, all of the following are true except A) gains on securities for a position held at least 12 months are not taxable. B) long-term gains are taxed at more favorable long-term rates. C) capital gains are associated with the sale of securities and other real assets. D) short-term gains are taxed at less favorable ordinary income tax rates.

Investment income, which includes capital gains realized on securities positons, is taxable. Depending on how long a security was held, the gains might be taxable at the investor's ordinary income tax rate (for short-term gains) or at a more favorable long-term rate if the position was held for longer than 12 months. LO 8.e A

Which of the debt issued listed here would produce tax-free interest at all levels? A) City of San Juan, Puerto Rico, general obligation bond B) Piute County, Utah, general obligation bond C) City of San Francisco, California, general obligation bond D) 30-year T-bond

Issues from a territory of the United States (like Puerto Rico) produce interest that is tax free at the federal, state, and local level. Municipal bond interest is tax free at the federal level and tax free at the state level if the bondholder is a resident of the same state as the issuer. Treasury issues are taxed at the federal level. LO 3.g A

During the course of a day a customer makes six separate cash deposits at six different branches totaling over $11,000. This may be an example of A) integration, and an SAR should be filed. B) structuring, and a CTR should be filed. C) application, and an SAR should be filed. D) extraction, and a CTR should be filed.

It appears that the customer is structuring deposits to avoid attention being drawn to the amount of cash he is depositing. This activity is called structuring. As the transactions exceed $10,000 in a day of currency, your firm would file a currency transaction report with FinCEN. LO 11.b B

An example of securities that are established by states to provide other government entities such as cities, towns, school districts or state agencies with a short-term investment vehicle to invest funds include A) money market instruments. B) bond anticipation notes (BANs). C) tax anticipation notes (TANs). D) local government investment pools (LGIPS).

LGIPs are established by states to provide other government entities within its borders such as cities, counties, school districts or other state agencies with a short-term investment vehicle to invest funds. LO 5.e D

All of the following are examples of legislative risk except A) an environmental regulation enacted to require certain precautions be taken. B) a law that would either allow or eliminate a tax deduction. C) a luxury tax imposed on high-priced amenities such as automobiles or yachts. D) changes made to the tax code regarding income tax.

Legislative risk results from a change in the law. Changes to the tax code are the most common legislative risks. Regulatory risk comes from a change to regulations that might impact certain individuals or businesses. The imposition of environmental regulations is one such example. LO 7.a A

In order for a business entity to qualify as a limited partnership, the LP must have A) any number of general partners and no limited partners. B) at least one general partner and one limited partner. C) any number of limited partners only. D) no general partners and at least one limited partner.

Limited partnerships are required to have at least one general partner and one limited partner. LO 5.f B

Two benefits of owning preferred stock over common stock are A) priority for payment of dividends ahead of wages and taxes and liquidation priority over wages. B) priority over subordinate bonds at liquidation and of dividends. C) rising interest rates are a positive for market value and dividends are guaranteed. D) priority at liquidation and payment of dividends.

Liquidation and payment of dividends are two areas in which preferred stock has a benefit over common stock. Dividends are not guaranteed and rising interest rates are a negative. Preferred gets paid after debt, wages, and taxes. LO 3.a D

Which of the following has a right and is bullish on the stock? A) Short put B) Long call C) Long put D) Short call

Long calls have a right and are bullish. Short calls have an obligation and are bearish. Long puts have a right and are bearish. Short puts have an obligation and are bullish. LO 5.a B

Which of these does M2 measure? I. M1 II. M3 III. Term repos and jumbo CDs IV. Retail CDs and money markets A) II and IV B) I and III C) II and III D) I and IV

M1 is a component of M2 and adds retail CDs and money markets. M2 is a component of M3, plus term repos and jumbo CDs. LO 9.h D

M1 is a measure of the value of A) M2 plus retail CDs and money markets. B) retail CDs and money markets. C) cash and funds held in DDAs. D) cash, cash equivalents, and DDAs.

M1 is the tightest of the money supply measures, including only actual cash in circulation and funds in demand deposit accounts (DDAs) (e.g., checking and savings accounts). LO 9.h C

Currency held by the public, including checking accounts and time deposits less than $100,000, and money market mutual funds would best be described by economists as A) M3. B) M1. C) M2. D) M4.

M2 is M1 (currency held by the public including checking accounts) plus time deposits less than $100,000 and money market mutual funds. LO 9.h C

All of these enforce MSRB rules except A) the Securities Exchange Commission. B) the MSRB Enforcement Department. C) the Federal Reserve Board. D) FINRA.

MSRB has no enforcement department. FINRA and the SEC enforce MSRB rules for broker-dealers, and the Fed and the Office of the comptroller do so for banks. LO 10.f B

The actions taken by the Federal Reserve to manage the economy are based on which of the following economic theories? A) Dickensian theory B) Keynesian theory C) Monetary theory D) Sharpe theory

Monetary theory (or monetarist theory) is the theory that guides the Federal Reserve in management of the money supply. LO 9.g C

Within the money supply, which of the following are part of M2 but not M1? A) Currency in circulation B) Demand deposits at S&Ls C) Money market mutual funds D) Checking accounts at commercial banks

Money market funds are part of M2 but not M1. M2 includes everything in M1, plus time deposits and money market funds. LO 9.h C

All of the following are considered money market instruments except A) exchange-traded funds (ETFs). B) commercial paper. C) banker's acceptances (BAs). D) negotiable jumbo certificates of deposit (CDs).

Money market instruments are short-term debt instruments. ETFs are equity securities. Remember that in order to be considered a money market security, the debt instrument should have one year or less to maturity. Banker's acceptances and commercial paper both have maximum maturities of 270 days; most negotiable jumbo certificates of deposit mature in one year or less. LO 3.f A

A money market security will have all of the following characteristics except A) a higher credit rating. B) high liquidity. C) a short term to maturity. D) a limited market.

Money market securities have short-term maturities. With little time left to default, they are considered to be highly liquid and, therefore, relatively safe. Safety or less risk equates to lower returns. Typically issued at a discount and maturing at face value, they generally make no regular interest payments. The difference between the discounted purchase price and the face value received at maturity represents their return. LO 3.f D

Portfolio diversifying might be used to reduce which of the following risks? A) Interest-rate risk B) Inflation risk C) Business risk D) Market risk

Nonsystematic risks can be reduced using diversification. These would include business, financial, credit, and liquidity risk (among others). Market, inflation, and interest-rate risks are types of systematic risks that are considered nondiversifiable because they impact all investments and, therefore, cannot be diversified away or mitigated simply by diversifying. LO 7.b C

The minimum initial requirement when purchasing 100 shares at $12 in a new account would be A) $600. B) $360. C) $2,000. D) $1,200.

Normally it would be 50% of the purchase price or $2,000, whichever is greater, but the required deposit is never more than 100% of the purchase price in a long account. LO 6.g D

A customer says they have a diversified portfolio of notes and bonds. This means their portfolio consists primarily of A) hedge funds. B) limited partnerships. C) debt instruments. D) equity securities.

Notes and bonds are types of debt and the term is often used generically to represent a debt securities. LO 3.a C

In which of the following accounts would the use of margin always be prohibited? A) Corporate accounts B) Individual retirement accounts C) Fiduciary accounts D) Partnership accounts

Of those listed, only qualified retirement accounts, such as IRAs, prohibit the use of margin. As long as the use of margin is not listed as being restricted, it is allowed in both corporate and partnership accounts, and as long as the use of margin is specifically listed as being allowed, a fiduciary account may do so. LO 6.g B

After the issuer files a registration statement with the Securities and Exchange Commission (SEC), the time known as the cooling-off period begins. This allows a registration to become effective as early as A) 40 calendar days after the date the SEC has received it. B) 20 calendar days after the date the SEC has received it. C) 40 business days after the date the SEC has received it. D) 20 business days after the date the SEC has received it.

Once the registration statement has been received by the SEC, a cooling-off period begins and it must last at least 20 calendar days. This allows the registration to become effective as early as 20 calendar days after the date the SEC has received it. LO 1.b B

All of the following characteristics are true of securities issued by the Government National Mortgage Association except A) they pay monthly. B) they generate tax-free interest. C) they are backed by the federal government. D) they are called pass-through securities because the payments are made up of both interest and principal.

GNMA interest is fully taxable. All the other statements are true. LO 3.e B

Skye purchased 100 shares of Moreno, Inc., for $20 a share. One year later, she sold the shares for $21 dollars. Over the year, Moreno paid a $0.25 quarterly dividend. What was Skye's gain or loss and how much investment income did she earn? A) $1 gain and $1 in income B) Cannot be determined from this information C) $2 total gain D) $2 in income

Gains are derived from opening and closing trades buy and sell in this example). She bought at $20 and sold for $21, so there is $1 in gain. She collected four quarterly dividends for $0.25 each, so a total in $1 in investment income. LO 8.d A

Which of the following would not be considered ordinary income for tax purposes? A) Dividends on common stock B) Gains gotten from the sale of securities C) Rents from income properties D) Salary and commissions

Gains gotten from the sale of securities is an example of capital gains for tax purposes. All the others are considered ordinary income. LO 8.d B

If an arbitration agreement has not been signed, under which two of the following circumstances would a dispute between a Financial Industry Regulatory Authority (FINRA) member firm and a retail customer go to arbitration? I. The dispute cannot otherwise be resolved to the satisfaction of both parties. II. The customer requests that the dispute go to arbitration. III. The amount under dispute is less than $50,000. IV. The FINRA director of arbitration so rules. A) III and IV B) I and II C) II and IV D) I and III

Generally speaking, a broker-dealer firm has the customer sign a binding arbitration agreement before opening the account. In the absence of such an agreement, if a dispute cannot otherwise be resolved, the dispute can go to arbitration if the customer wishes it so. LO 10.e B

Which of these would not be fully covered by SIPC insurance? A) Holding $250,000 in index funds, $200,000 in Treasury bonds, and $50,000 in gold B) Holding $300,000 in a money market and $200,000 in mutual funds C) Holding $250,000 in cash and $250,000 in the Windmill Commodity ETF D) Holding $400,000 in junk bonds and $100,000 in cash

Gold is not a security and is not covered by SIPC. Money markets, ETFs, mutual funds, and junk bonds are all types of securities. LO 10.c A

Your client bought 100 shares of ABC at $50 per share and later received a 10% stock dividend. What is her new cost basis per share? How many shares does she now have? A) 100 shares at 45.45 B) 110 share at $45.45 C) 100 shares at $55.00 D) 110 shares at $55.00

Her original cost basis was 100 × $50 = $5,000. She now has 110 shares still worth $5,000. $5,000 divided by 110 equals $45.45. LO 2.h B

Which of the following activities would be handled by a carrying firm and not an introducing firm? A) Answering customer inquiries B) Soliciting trades C) Issuing statements for customer accounts D) Holding custody of a customer's securities

Holding customer assets is a task handled by a carrying firm rather than an introducing firm. All of the other activities listed here are common to all broker-dealers. LO 1.e D

After the issuer files a registration statement with the Securities and Exchange Commission (SEC), the time known as the cooling-off period begins. This allows a registration to become effective as early as A) 40 calendar days after the date the SEC has received it. B) 20 calendar days after the date the SEC has received it. C) 40 business days after the date the SEC has received it. D) 20 business days after the date the SEC has received it.

Once the registration statement has been received by the SEC, a cooling-off period begins and it must last at least 20 calendar days. This allows the registration to become effective as early as 20 calendar days after the date the SEC has received it. LO 1.b B

Efforts to push up the price of stock one owns by soliciting buy orders from customers so that the stock owned can be sold later at a higher price is a violation commonly known as A) marking the open. B) churning. C) pump and dump. D) marking the close.

One form of securities fraud where an effort is made to push up the price of a stock that is currently owned by soliciting buy orders so that the stock can be sold later at a higher price is known as a pump and dump scheme. LO 11.c C

Which of these features does the Roth IRA include? I. There are no minimum required distributions after age 72 with a Roth IRA. II. There are higher contribution limits to a Roth IRA than to a traditional IRA. III. Withdrawal of earnings in the Roth IRA may be made without any taxation as long as a Roth IRA has been open for a minimum of one year and the participant is age 59. IV. There is the ability to contribute to both a Roth IRA and a traditional IRA. A) II and I B) II and III C) I and IV D) I and III

One of the primary benefits to the Roth IRA is that reaching age 72 does not trigger the required minimum distributions found in other retirement plans. Probably the biggest benefit is that all earnings grow tax deferred, and may be withdrawn free of any tax, as long as there has been an open Roth IRA for at least 5 years AND the participant is at least 59½. One may contribute to both types of IRA, but the combined contribution may not exceed that annual maximum for a single plan. LO 6.e C

Under the Uniform Securities Act of 1956, who is responsible for enforcement of state securities regulations? A) The Securities Exchange Commission B) The State Securities Rulemaking Board C) The Financial Industry Regulatory Authority D) The Administrator

One of the provisions of the USA is that each state have a state securities administrator (the administrator). We made up the State Securities Rulemaking Board. LO 10.d D

Should a member firm or an associated person be found in violation of Financial Industry Regulatory Authority (FINRA)'s Conduct Rules, a number of sanctions may be imposed. However, under the Code of Procedure, FINRA may not A) impose a prison sentence on the violator. B) bar an associated person from the industry forever. C) censure the violator. D) issue a fine to a member firm or associate.

Only a court can issue a prison sentence. Each of the remaining answer selections, bar, fine, and censure, are sanctions FINRA could impose under the Code of Procedure. LO 10.e A

Which of these would not affect the NAV per share of a mutual fund share? A) The fund receives a dividend from one of the portfolio stocks. B) The portfolio's market value undergoes a large increase. C) The fund pays its monthly operating expenses like utility bills. D) Portfolio securities had to be sold for a big capital loss.

Selling securities out of the portfolio, whether for a gain or a loss, simply replaces the securities with an equivalent amount of cash, leaving the NAV per share unchanged. The other choices involve changes in net assets with no accompanying change in the number of shares outstanding, which would change the NAV per share. LO 4.c D

Which of the following transactions, if any, cannot be done in a cash account? A) Buy 100 ABC to open B) Any of these could be done in a cash account C) Sell 100 ABC to close D) Sell 100 ABC to open

Selling to open (a short sell) can only be done in a margin account. It cannot be done in a cash account. LO 6.g D

A signature endorsement on a stock certificate would not be required for good delivery if the shares were A) are in custodial name. B) in trust name. C) held in safe keeping. D) held in street name.

Shares held in street name are normally registered to the broker-dealer, who hold them on behalf of the client. As these shares are likely only in electronic form no signature is required to transfer them. LO 1.i D

Who is responsible for filing a U5 with the Central Registration Depository (CRD) following termination? A) FINRA B) The member firm C) The registered person D) The depository trust company

Should a person registered with a member firm resign or be terminated, the member must file Form U5 with the Central Registration Depository (CRD) within 30 days of termination date. LO 12.a B

Mutual funds that market directly to the public, using no underwriter and charging no sales charge, are called A) free-sale funds. B) charge-free funds. C) loaded funds. D) no-load funds.

Some funds distribute their own shares without using an underwriter and, hence, have no need to levy a sales charge. Because sales charges are also called sales loads, such funds are known as no-load funds. LO 4.b D

Sovereign risk is the risk A) that interest rates decline in several countries simultaneously. B) of losing all one's investment due to a change in tax laws. C) that a dollar earned today will not be able to purchase the same goods or services it can now in the future. D) that a country will default on its commercial debt obligations.

Sovereign risk is when a country is at risk of defaulting on its commercial debt obligations. When this occurs, the impact is felt on financial markets worldwide. LO 7.a D

A toy company is experiencing sudden strong demand for a game. Purchasing this company's stock may prove profitable in the short run. This company's stock might best be termed as A) a growth. B) a short sale. C) a special situation. D) a cyclical.

Special situations arise when a company shows unusual profit potential resulting from nonrecurring circumstances. These situations might include new management, the discovery of a valuable natural resource on corporate property, patents pending, or the introduction of a new product. LO 9.c C

A supply-side approach to fiscal policy will use all of these tools except A) decreasing tax rates on business entities. B) providing tax credits to small business. C) personal income tax rebates. D) decreasing government regulatory costs.

Supply-side fiscal policy seeks to create a better environment for business to thrive. The end goal is a growing economy that creates jobs. Sometimes called trickle-down economics, the emphasis is on the business side much more than the consumer side. LO 9.e C

Risk that is difficult to reduce by diversification within an asset class is called which of these? A) Nonsystematic risk B) Systematic risk C) Financial risk D) Unsystematic risk

Systematic risk affects an entire asset class in the same way, so diversification does little to reduce systematic risks. Market and interest rate risk are examples. Unsystematic and nonsystematic risk are different terms for the same class of risk. Financial risk is a specific nonsystematic risk. LO 7.a B

Systematic risk is A) may be significantly reduced by diversification within an asset class. B) associated with equity investments. C) associated with debt instruments. D) not reduced by diversification within an asset class.

Systematic risk is risk that remains despite diversification. Examples include market risk and interest rate risk. LO 7.a D

Deposits received in currency for amounts over $10,000 in a day would require the firm to report the transaction in how many days on what form? A) CTR within 30 days B) SAR within 15 days C) SAR within 30 days D) CTR within 15 days

The Bank Secrecy Act requires firms to report on a CTR any currency received in the amount of more than $10,000 on a single day, within 15 days. LO 11.b D

Which of the following statements would describe the Fourth Market? A) A market for institutional investors in which large blocks of stock, both listed and unlisted, trade in transactions unassisted by broker-dealers B) These transactions take place through electronic communications networks (ECNs). ECNs are open 24 hours a day and act solely as principals C) These transactions take place through electronic communications networks (ECNs) which are open during normal trading hours and act solely as principals D) The after-hours market

The Fourth Market is a market for institutional investors in which blocks of stock trade through ECNs that are open 24 hours a day acting as agents. LO 1.d A

All of the following terms and phrases apply to the buy side of the options contract except A) pays the premium. B) exercises the contract. C) has a right. D) wants the contract to expire.

The buyer of the contract pays the premium and loses it if the contract expires. The seller receives the premium and keeps it if the contract expires. The buyer has a right to exercise the contract. The seller has an obligation if the buyer decides to exercise. LO 5.a D

When a bond is purchased at a discount the current yield will be A) the same as the nominal rate. B) higher than the coupon rate. C) lower than the fixed rate. D) lower that the stated rate.

The coupon rate, the stated rate, the fixed rate, and the nominal rate all mean the same thing. It is the amount the bond will pay each year. On a discount bond the current yield is always higher than the coupon rate. LO 8.a B

In the last year, a customer made the following four transactions: Purchased 1,000 shares of ABC stock for $67 per share Purchased 500 shares of DEF stock for $45 per share Sold 1000 shares of ABC stock for $50 per share Sold 500 shares of DEF stock for $55 per share Which of these is the result of the year's transaction? A) $12,000 reduction in ordinary income B) $12,000 in capital gains C) $3,000 reduction in ordinary income and $12,000 in gains D) $3,000 reduction in ordinary income and $9,000 in carry-forward losses

The customer realized a $17,000 loss on ABC and a $5,000 gain on DEF for a net loss of $12,000. The customer can use $3,000 to reduce ordinary income leaving $9,000 in losses to carry forward. LO 8.e D

Your customer is a resident of a state with no income tax. They want an income-producing investment that produces tax-free income for them. Which of the following meets the customer's needs? A) Only a municipal bond issued by the customer's home state B) A Treasury bond C) A GNMA certificate D) A municipal bond issued from any state

The customer's state of residence has no income tax, so any municipal bond would meet the customer's need. A municipal bond issues by the customer's home state is not the correct answer only because it states "only," which is not true. The customer would pay federal income tax on the T-bond and the GNMA. LO 3.g D

A customer called his registered representative to place a trade to buy 100 shares of ABC. The customer wants to put a limit on the order, but is unsure what would be an appropriate price. At the suggestion of the registered representative, the customer enters the order with a limit of $30. This trade was A) solicited. B) not held. C) unsolicited. D) discretionary.

The customer, independent of the registered representative, placed the order, making it unsolicited. While the rep did advise on what an appropriate limit price would be, the customer ultimately placed the order instructions with the limit, and would not be considered discretionary. LO 6.h C

Which of the following interest rates is set by the FRB? A) Prime rate B) Federal funds rate C) Broker call loan rate D) Discount rate

The discount rate is the rate of interest the Federal Reserve Bank (FRB) charges member banks for short-term loans. The other rates are set by the bank making the loan. LO 9.i D

During the cooling-off period the disclosure document that may be delivered to interested parties is called the A) preliminary prospectus. B) final prospectus. C) cool off period prospectus. D) summary prospectus.

The document available during the cooling-off period in the preliminary prospectus, also called a red herring. LO 1.b A

All of these dates are set by the board of directors of a corporation except A) the payable date. B) the ex-dividend date. C) the record date. D) the declaration date.

The ex-date is set by the market center (i.e., an exchange), or is set by FINRA if it is an over-the-counter traded security. LO 4.d B

When the Options Clearing Corporation (OCC) receives a notice to exercise, it will assign that notice to A) the party short the contract. B) the party long the contract. C) a short broker-dealer. D) a long broker-dealer.

The exercise and assignment process is as follows: A long customer notifies its broker-dealer (long broker-deal). The long broker-dealer notifies the OCC then assigns the short broker-dealer, who will then in turn assign its short customer. LO 5.c C

On March 3, the board of directors of Seabird Airlines declares a $0.20 per share dividend payable to holders of record as of March 30. Seabird stock jumped from $35 per share to $40 per share on the news. The current yield of Seabird stock is A) 2.00%. B) 5.00%. C) 2.25%. D) 0.50%.

The formula is (quarterly dividend x 4) / current market value. (0.2 x 4) / 40 = (.8) / 40 = .02 (2%) LO 8.a A

Which of the following are elements of insider trading? I. The information is material, nonpublic information. II. The information is nonmaterial, nonpublic information. III. It is illegal to trade on insider information. IV. It is illegal to possess insider information. A) II and IV B) II and III C) I and IV D) I and III

The information must be of a material nature. In other words, it would affect the price of the security if it was known, and a trade must occur for there to be a violation. LO 11.d D

The inverse relationship between interest rates and bond prices helps in understanding that interest rate fluctuations are A) a systematic risk for bonds. B) an unsystematic risk for bonds. C) the difference between credit and financial risk. D) an example of regulatory risk.

The inverse relationship illustrates a significant systematic risk for bonds and other fixed-income investments. When rates move up all bonds move down. Credit and financial risk are largely synonymous terms. Interest rate risk is not a regulatory risk. LO 7.a A

Your investor invests in a mutual fund at an offering price slightly higher than the fund's net asset value (NAV). Your investor is likely purchasing what share class? A) A shares B) D shares C) B shares D) C shares

The investor is paying the sales charge up front, so they are most likely buying Class A shares. There are no D shares on the exam. LO 4.b A

An investor notices that a bond purchased several years ago at 95 is now priced at 90. The investor sells the bond for 90, then immediately repurchases it for 90. This action is known as A) pegging. B) marking the close. C) a wash sale. D) matched orders.

The investor's intent with this wash sale is to declare a $50 capital loss without changing positions on the bond. Immediate repurchase is not illegal, but it precludes declaring the loss for tax purposes. The investor must wait at least 30 days before buying the bond back, or the loss will be disallowed. LO 8.d C

Raising funds is generally accomplished by corporations through the issuance of stock (equity) or bonds (debt). This is done in A) the secondary market. B) the funding market. C) the capital market. D) the currency market.

The issuance of stock or bonds by corporations to raise new funds takes place in the capital market. LO 1.a C

Which of the following investment companies has an actively managed portfolio? A) Debt fixed unit investment trust (UIT) B) Equity fixed unit investment trust (UIT) C) Closed-end company D) Face-amount certificate company

The portfolios of both face-amount certificate companies and UITs are nonmanaged. The closest they come to management is when the securities to make up the portfolio are selected. After that, the portfolio does not change. Closed-end companies have an investment adviser who actively manages the portfolio, buying and selling securities. LO 4.a C

MAS Corporation has enjoyed an extremely profitable year. It has been determined that those owning the MAS 4% preferred, participating to 6% preferred shares, will receive the full participating dividend. The participating shareholders will receive an additional dividend of A) 2%. B) 6%. C) 10%. D) 4%.

The stated MAS preferred dividend is 4%, participating up to 6%. In this year, when it has been determined that they should receive the full participating dividend, they will receive the additional participating 2%. LO 2.d A

A registered representative opens a new options account for a customer. In which order must the following actions take place? I. Obtain approval from the branch manager II. Obtain essential facts from the customer III. Obtain a signed options agreement IV. Enter the initial order A) II, I, IV, III B) I, II, III, IV C) II, I, III, IV D) I, II, IV, III

The steps or order of events to open an options account would occur in the following order: obtain essential facts about the customer, give the customer an options disclosure document (ODD), have the manager approve the account, enter the initial order, and have the customer sign the options agreement within 15 calendar days. LO 5.c A

A 6% corporate bond trading on a 7% basis is trading A) at a discount. B) with a coupon rate below 6%. C) with a current yield above 7%. D) at a premium.

The term a 7% basis means that the YTM is 7%. YTM is higher than the coupon rate (6%), so the bond trades at a discount. Current yield must be between the coupon rate and the YTM. LO 3.a A

The Investment Company Act of 1940 classified all the following as investment companies except A) face-amount certificates. B) management companies. C) private investment companies. D) unit investment trusts.

The three classifications established under the Investment Company Act of 1940 are face-amount certificates, unit investment trusts, and management companies (open and closed-end funds). Private investment companies do not come under the Act of 1940. LO 4.a C

Your customer is quite nervous about the stock market but expresses his belief that equities are still the place to save for retirement over the long term. He places a trade for 500 shares of an equity index fund. Overall your customer is likely A) a bear. B) a bull. C) unsure of where the market is going. D) a tiger.

This customer has expressed confidence in stocks and has invested money to back up his belief. He is not confused at all. He recognizes that the equities can produce nerve-wracking consequences but have the best record of long-term growth, He is a bull. LO 1.f B

A central, physical, marketplace where securities are traded through a designated market maker is A) the pit. B) the OTC. C) the third market. D) an exchange.

This description most aligns with the exchange model where securities are traded in a central location through a designated market maker. LO 1.d D

Kamron owns a diversified portfolio of stocks. The portfolio holds 58 different stocks that are diversified by market capitalization and sector as well as industry. When the stock market entered a significant downward correction Kamron's portfolio also dropped. This is an example of which of these? I. Market risk II. Business risk III. Systematic risk IV. Unsystematic risk A) II and III B) I and III C) II and IV D) I and IV

This example represents a market risk and a systematic risk. LO 7.a B

Your customer purchases 200 shares of Seabird Airlines (the ticker is SBRD) at $30 a share in a cash account. Under Regulation T, the Federal Reserve has set the initial margin requirement at 50%. How much does your customer need to deposit for this trade? A) $6,000 B) $3,000 C) $2,000 D) $1,500

This is a cash account. There is no margin borrowing, so 100% of the trade's value must be deposited. 200 shares at $30 = $6,000. LO 6.g A

The economic indicator that reflects activity of U.S. entities without regard to where the activity takes place is A) GNP. B) CPI. C) FUN. D) GDP.

This is a description of gross national product. GDP measures activity within the U.S., regardless if it is domestic entity or not. CPI is the primary inflation measure. FUN is just a word. Do not pick something you do not recognize just because it is unfamiliar. LO 9.b A

The part of the federal government that acts as an independent central bank in order to manage monetary policy is the A) Comptroller of the Currency. B) Department of the Interior. C) Treasury Department. D) Federal Reserve.

This is a description of the duties of the Federal Reserve. LO 9.g D

Deanfield and Chatham Investments, LLC has recently registered as a broker-dealer with the SEC and been accepted as a FINRA member firm. They update their website to prominently display this fact at the top of their site and include large images of both organizations' logos. This is A) not allowed because it is a misrepresentation. B) not allowed because it violates principals of marketing. C) allowed so long as it is accurate. D) allowed with proper disclaimers.

This is a misrepresentation. Even if only stating a fact, the prominence suggests some greater affiliation or approval. Deanfield and Chatham is about to get a rather terse letter from these regulators, or worse. LO 10.a A

Ron buys 522 shares of Narcissus, Inc., common stock in a cash account on Monday, March 19. He deposits cash in the account sufficient to cover the trade on March 20. How soon would he be able to withdraw those shares? A) Wednesday, March 21 B) Tuesday, March 20 C) Friday, March 23 D) Thursday, March 22

This is a regular way settlement of a corporate security (T+2). The trade is fully paid for so the customer may request the shares be moved on or after settlement on Wednesday, March 21. LO 1.i A

If a fund sponsor allows an investor to move funds from one fund to another within its fund family, this is called A) a reinvestment right. B) a right of accumulation. C) a 12b-1 waiver. D) an exchange privilege.

This is an example of an exchange (or conversion) privilege. LO 4.b D

The risk that a stock will not appreciate in value due to poor cash flow is an example of A) sales risk. B) systematic risk. C) nonsystematic risk. D) market risk.

This is an issue that applies to just this issuer; a nonsystematic risk. LO 7.a C

The personal data assistant maker Hands-On, Inc., spent considerable resources developing a new product that used a unique user interface. Unfortunately, the interface proved a failure and was quickly surpassed by a competitor's system. The firm's stock fell as a result. This is an example of A) nonsystematic risk. B) sales risk. C) market risk. D) systematic risk.

This is an issue that applies to just this issuer—a nonsystematic risk. LO 7.a A

What government organization maintains the Specially Designated Nationals List? A) FINRA B) FBI C) OFAC D) FinCEN

This is one of the functions of the Office of Foreign Asset Control (OFAC), a division of the Treasury that enforces economic and trade sanctions. LO 11.b C

Evan is a 75-year-old customer with $100,000 to invest. He would like the money to generate additional income. He relates that he intensely hates paying taxes and dislikes the government in general. He is, however, interested in tax-free municipal bonds. All of these are important to gather before making a recommendation except A) why he hates the government. B) his liquid net worth. C) the makeup of his portfolio. D) his current tax bracket.

Though it might be interesting to find out why he hates the government, the others are all basic points of suitability. LO 3.d A

The SEC has regulatory authority over all of these entities except A) exchanges. B) FINRA. C) municipalities. D) Securities Information Providers (SIPs).

Though the SEC does oversee the MSRB, neither the SEC nor the MSRB regulates municipalities in the U.S. The SEC does oversee FINRA and the exchanges. LO 10.a C

The Securities Exchange Commission was authorized by A) the Securities Act of 1933. B) the Securities Exchange Act of 1934. C) the Securities Exchange Act of 1933. D) the Securities Act of 1934.

Though the Securities Act of 1933 required registration for new securities with the SEC, the SEC was not authorized until passage of the Securities Exchange Act of 1934. LO 10.a B

A customer purchases BigCo common stock at $30 per share. The stock pays a $0.30 quarterly dividend. After holding the position for one year, the customer sells the stock for $31.80 per share. What is the customer's total return? A) 6.5% B) 7% C) 10% D) 4%

To calculate total return you would add the dividends collected over the period and any capital gains (or subtract capital losses). The customer collected $1.20 in dividends (0.3 × 4 quarters) and realized a gain of $1.80 ($1.20 = 1.80 = $3.00). Divide the result by the purchase price to determine the total return. $3/$30 = 0.10 (10%) LO 8.c C

A customer purchases LMNO common stock at $60 per share. The stock pays a $0.45 quarterly dividend. After holding the position for one year, the customer sells the stock for $61.20 per share. What is the customer's total return? A) 5% B) 4% C) 2% D) 3%

To calculate total return you would add the dividends collected over the period and any capital gains (or subtract capital losses). The customer collected $1.80 in dividends (0.45 × 4 quarters) and realized a gain of $1.20 ($1.20 = 1.80 = $3.00). Divide the result by the purchase price to determine the total return. $3/$60 = 0.05 (5%) LO 8.c A

Which of the following firms would not normally be part of an initial public offering (IPO)? A) Underwriters B) Investment bankers C) Third-market dealers D) Broker-dealers

To facilitate a public offering, issuers will use the services of investment bankers and broker-dealers (also called underwriters) of the securities. Third-market dealers function in the over-the-counter market (secondary market), not the primary market. LO 1.a C

Drew purchased 100 shares of Moreno, Inc., for $20 a share. One year later, he sold the shares for $21 dollars. Over the year, Moreno paid a $0.25 quarterly dividend. What is Drew's total return? A) $2. B) 5%. C) $1. D) 10%.

Total return includes any income the investment produces; it is also expressed as a percentage, not dollars. The formula is as follows: [(sales proceeds - cost basis) + income)] / cost basis. Using the formula here, the calculation is as follows: [(21 - 20) + (4 × .25)] / 20 = (1 + 1) / 20 = 2 / 20 = .1 (or 10%). LO 8.c D

A variable annuity (VA) would be best for which of the following customer objectives? A) Additional retirement income for a 70-year-old customer B) Saving for college tuition for a customer's 14-year-old child C) Saving for a down payment on a first home for a young couple D) Current income for a 50-year-old customer

Variable annuities are designed to provide a supplement to retirement income that may keep pace with inflation. Income withdrawn from a VA is subject to income tax and a 10% penalty for owners under age 59 ½. VAs often have contingent deferred sales charge (CDSCs) that make them a poor choice for short-term investors. LO 4.g A

All of the following points are aligned with recommending a variable annuity (VA) except A) the investor wants a supplement to their retirement income. B) the investor is comfortable with market risk. C) the investor will fund the investment with assets earmarked for retirement from a 401(k). D) the investor will fund the investment with assets earmarked for retirement from a non-qualified money market account.

Variable annuities are designed to provide a supplement to retirement savings. Funding for a VA should come from available cash not from a qualified plan like a 401(k) or an IRA. VA investors should be comfortable with market risk. LO 4.g C

Your customer, Eleanor, purchased an InDebt, Inc., 5% debenture at a price of 94. It matures in 12 years. What is the yield to maturity? A) 5.32 B) 4.69 C) 5 D) 5.73

You do not have to calculate YTM for this problem. You could if you really wanted to, but it is not necessary for the question. You do need to recall the bond inverse relationship chart. The bond is trading at a discount so the YTM must be higher than the coupon of 5%; that eliminates two responses. Note that YTM is higher than current yield, and that you do need to calculate CY. The bond's annual interest divided by the price (50/940) is 5.32% (the CY). Only one response is higher than 5.32% LO 3.a D

An institutional customer, such as a hedge fund, utilizes the services of a broker-dealer who provides custody of securities, as well as other back-office functions, while allowing the customer to establish relationships with other broker-dealers for the purpose of executing orders. This account would be known as a A) clearing account. B) fully-disclosed account. C) prime account. D) self-clearing account.

A broker-dealer that provides custody of securities and other back-office functions but allows the customer to maintain relationships with other broker-dealers who will provide execution services is known as a prime broker-dealer. The account, known as a prime account, is maintained with the prime broker rather than with any executing brokers. LO 1.e C

SKRAM Corporation is appealing directly to the shareholders of IDNIC Corporation to acquire shares of IDNIC stock. This appeal is best described as A) a buy back with SKRAM the bidder company. B) a hostile takeover with IDNIC the target company. C) a sell-off with IDNIC the target company. D) an acquisition.

A hostile takeover is accomplished when the buyer (SKRAM) goes directly to the target (IDNIC) company's shareholders bypassing the board of directors or management. LO 2.i B

Which of the following statements is correct concerning currency risk when investing in an American depositary receipt (ADR)? A) U.S. investors are protected from currency risk by the underlying foreign corporation. B) Currency risk is eliminated because the securities are dollar denominated. C) Currency risk is still a factor when purchasing an ADR. D) U.S. investors are protected from currency risk by the depositary bank.

ADRs are issued and pay dividends in U.S. dollars eliminating the complications of currency conversion. However, ADRs are still subject to currency risk. Why? The company pays dividends in its home currency, and the issuing bank pays out those dividends in U.S. dollars. When the exchange rate changes, the amount these dividends (in U.S. dollar terms) will fluctuate as well. Also, the value of the ADR itself will rise and fall with the value of the underlying foreign stock which is partially due to currency swings. LO 2.e C

The primary purpose of American depositary receipts (ADRs) is to facilitate the trading of A) foreign stocks in U.S. markets. B) domestic stocks in both foreign and domestic markets. C) U.S. stocks in foreign markets. D) foreign stocks in both domestic and foreign markets.

ADRs facilitate the trading of foreign stocks in U.S. markets. LO 2.e A

Which of the following preferred stocks' price would remain most stable in an environment of changing interest rates? A) Participating preferred B) Adjustable rate preferred C) Straight preferred D) Nocumulative preferred

Adjustable rate preferred dividend resets when interest rates change so the price remains stable. LO 2.d B

All else being equal, which of the following preferred would pay the highest dividend? A) Participating preferred B) Straight preferred C) Callable preferred D) Cumulative preferred

Callable preferred is a benefit to the issuer—not the investor—so callable has to pay a higher dividend than the others because the other features are neutral or benefit the investor. LO 2.d C

All of the following are market centers within the secondary market except A) over-the-counter. B) capital. C) third market. D) exchanges.

Capital markets is a term that covers all the markets used in capital formation. The others are all parts of the secondary markets in the United States. LO 1.d B

DEF Growth Fund has net assets of $330 million and liabilities of $15 million. The fund has 10 million outstanding shares. What is the fund's current net asset value (NAV) per share? A) $32.00 B) $34.50 C) $33.00 D) $31.50

NAV is calculated by dividing the net assets of the fund by the number of outstanding shares. In this question, the net assets are given; the liabilities are already in the figure. The math is 330 million / 10 million = $33 per share. LO 4.c C

In the dividend disbursement process three of the four critical dates are determined by the board of directors (BOD) but one is determined by either Financial Industry Regulatory Authority (FINRA) for OTC stocks or the exchange for listed stocks. Which one is it? A) payable B) ex-dividend C) declaration D) record

Declaration, record, and payment dates are determined by the board of directors (BOD), but FINRA, or the exchange, determines the ex-dividend date. LO 2.b B

Rank the following government-issued securities from shortest to longest maturity. A) Treasury bills, bonds, and notes B) Treasury bills, notes, and bonds C) Treasury bonds, notes, and bills D) Treasury notes, bills, and bonds

For U.S.-government-issued securities, T-bills have the shortest maturities (one year or less), T-notes have longer maturities (2-10 years), and T-bonds have the longest maturities (greater than 10 years, up to 30 years). LO 3.e B

A customer owns cumulative preferred stock (par value of $100) that pays an 8% dividend. The dividend has not been paid this year and was missed in the two previous years. If the company wants to pay a dividend to common shareholders, how much must the company pay this customer per share first? A) $8 B) $24 C) $0 D) $16

If the company is going to pay a common stock dividend, it must pay the preferred dividends first, including all dividends in arrears (missed). There are $16 due in back dividends for the two years missed, in addition to $8 this year, for a total of $24. LO 2.d B

A shareholder owns preferred shares that allow for the possibility of receiving more than the stated dividend. This type of preferred share would be known as A) adjustable. B) convertible. C) participating. D) callable.

In addition to the fixed stated dividend, participating preferred stock offers its owners the possibility of receiving a share of corporate profits that remain after all dividends and interest due other securities are paid. LO 2.d C

Federal, state, and local income tax would be due on the interest from which of the following issues? A) Jersey City, New Jersey, general obligation bond B) Grant anticipation note issued by Ogden, Utah C) 90-day Treasury bill D) FNMA certificate

Interest from an FNMA certificate is taxed at all levels. Municipal bonds are tax free at the federal level. Treasury issues are tax free at the state level. LO 3.g D

Which of the following statements with regard to the issuance of securities is true? A) Once a registration statement has been filed with the Securities and Exchange Commission (SEC) it should be expected that the securities could be sold to the public within two business days. B) The Securities Act of 1933 provides criminal penalties for fraud. C) The cooling-off period beginning when a registration statement is filed with the Securities and Exchange Commission (SEC) can't last longer than 20 days. D) While the Securities and Exchange Commission (SEC) is reviewing a registration statement for a new offering of securities, the underwriters are permitted to solicit and accept orders for the securities from the public.

The Securities Act of 1933 (also known as the Paper Act, Full Disclosure Act, New Issues Act, Truth in Securities Act, and Prospectus Act) ensures that the investing public is fully informed about a security and its issuer when the security is offered on the primary market. The act provides criminal penalties for fraud in the issuance of new securities. The SEC review period, known as the cooling-off period, must last a minimum of 20 days before the SEC releases the securities for sale to the public (effective date). Solicitations and the acceptance of orders may never occur before the effective date. LO 1.b B

Your customer purchased 300 shares of XYZ stocks six months ago and sold the shares last week. The actions your customer took in relation to XYZ were to A) buy long and sell short. B) buy long and sell long. C) buy short and sell long. D) buy short and sell short.

The purchase of the stock is a long buy. The subsequent sale of the long position is a long sale. LO 1.g B

ABC Corporation's board of directors declared a $0.25 per share dividend on Wednesday, June 15. The dividend will be paid to shareholders of record on Tuesday, July 5. The dividend will be sent to shareholders on Tuesday, July 26. What is the last day to purchase the stock and receive the dividend, assuming a regular way settlement? A) June 30 B) July 3 C) July 2 D) July 1

This is a potent example of a difficult question that requires you to slow down and think it through. The ex-date for a corporation will be one business day prior to the record date. The record date is July 5. The day before is July 4, which is a holiday and not a business day. The days before that are Sunday and Saturday. The first business day before the ex-date is Friday, July 1. The last day to purchase the stock and qualify for the dividend is the day before the ex-date (June 30). The only holidays we have heard of appearing on the exam are July 4 and December 25. LO 2.b A

Under whose Social Security number is the custodial account established? A) The person who established the account for the minor, whether the parent or not B) The custodian's C) The parent, whether the parent is custodian or not D) The minor's

Assets in a custodial account are the minor's property, so the minor's Social Security number is used. LO 6.c D

An investor can take advantage of intraday price changes due to normal market forces when investing in which of these? I. Closed-end funds II. Exchange-traded funds III. Hedge funds IV. Open-end funds A) II and III B) I and IV C) III and IV D) I and II

Both closed-end funds and ETFs trade in the marketplace based upon supply and demand. Open-end funds use forward pricing and generally price only once per day (usually at the end of the trading day). Most hedge funds are organized as private investment partnerships and are considered illiquid. Some have minimum holding requirements known as lock-up provisions, and in that light, their interests do not reliably trade intraday. Information on ETFs can be found in Unit Five under Learning Objective 5.l. LO 4.a D

An investor is long a January 30 call at 2. Breakeven is A) 30. B) 32. C) 200. D) 28.

Breakeven for a long call is premium (2) plus strike price (30)—in this case, 32 points. The investor needs the stock to be above the breakeven point to make a profit. LO 5.a B

FinCEN is a bureau of what department within the federal government? A) Interior B) Treasury C) Defense D) Justice

Though it is an intelligence-gathering organization that watches for suspected criminal activity, FinCEN is part of the Treasury Department. LO 10.b B

The monthly unemployment figure is considered a A) leading indicator. B) dragging indicator. C) lagging indicator. D) coincident indicator.

Unemployment is a coincident indicator. There is no dragging indicator. LO 9.b D

Which of the following is true of income received from an Achieving a Better Life Experience (ABLE) account? A) The income is tax free. B) The income is tax deferred. C) The income is taxed on withdrawal. D) The income is taxed in the year received.

Income from an ABLE account is tax free to the beneficiary. LO 5.e A

Your customer calls you with a question. They tell you that they received a phone call from the bond desk telling them that they bought 20 bonds at 100. They want to know how much they paid for the bonds before any commission or other charges. You tell them A) $1,000. B) $20,000. C) $200,000. D) $2,000.

100 means they paid 100% of par ($1,000) per bond. They purchased 20 bonds, so the total amounts to $20,000. Note that the question asked how much they paid for the bonds, not the price per bond. LO 3.a B

Your customer, Jim, wants to deposit money into a 529 college savings plan for his great-niece Penelope. He states four reasons why he likes the 529 plan. Unfortunately, you need to tell him he is incorrect on one point. Which of his following points is not considered a feature of a 529 college savings plan? A) The money grows tax deferred. B) She has to use the money by the time she turns 30, so she will not be able to put it off too long. C) If she gets into a good prep school the money can be used for that as well as college. D) The growth can be tax free if used for qualified education expenses.

529 plans grow tax deferred and the funds may be withdrawn tax free if used for qualified education expenses. These plans may be used to fund secondary education (pre-college). There is no age limit on when the funds must be used. LO 6.d B

A CMO consists of A) an FNMA, FHLMC, and other mortgage backed securities. B) different sorts of nonmortgage debt. C) various government backed mortgages. D) bonds and money market

A Collateralized Mortgage Obligation is made up of different mortgage backed securities (including FNMA and FHLMC), not the mortgages directly. LO 3.e A

A CMO consists of A) an FNMA, FHLMC, and other mortgage backed securities. B) different sorts of nonmortgage debt. C) various government backed mortgages. D) bonds and money market instruments.

A Collateralized Mortgage Obligation is made up of different mortgage backed securities (including FNMA and FHLMC), not the mortgages directly. LO 3.e A

A member of the Federal Reserve System wanting to increase its reserves could do so by borrowing money from A) another FRB member bank at the prime rate. B) another FRB member bank at the discount rate. C) the Federal Reserve Board (FRB) at the federal funds rate. D) the Federal Reserve Board (FRB) at the discount rate.

A Federal Reserve Board member bank can increase its reserves by borrowing from the Federal Reserve Bank directly, or it can borrow from another FRB member bank. When borrowing from the FRB directly, a bank will pay the discount rate. When borrowing from another member bank, a bank will pay the federal funds rate. LO 9.h D

An officer of a financial firm has identified what might represent suspicious behavior on the part of a customer, involving more than $5,000. When must the firm file a suspicious activity report (SAR)? A) By the end of the business day B) Within 5 business days C) Within 30 calendar days D) Within 180 calendar days

A SAR must be filed with FinCEN within 30 calendar days of the firm becoming aware of the suspicious activity. The suspected parties may not be informed that they are the subject of an SAR. LO 11.b C

Your client, Soren Aland, buys a 4% XYZ corporate bond. If his current yield is 5%, he bought the bond A) above par. B) at par. C) at a discount. D) at a premium.

A bond purchased at a discount will have a current yield above the coupon rate. LO 8.a C

Shelby Bogden, your client, purchased a 6% corporate bond with a current yield of 5%. The bond was purchased at A) below par. B) a premium. C) par. D) a discount.

A bond purchased at a premium will have a current yield below the coupon rate. LO 8.a B

Dewey Cheatham, a registered representative for Great Plains Securities, was found by the SEC to have committed fraud by stealing from the accounts of several elderly customers of the firm. The SEC has also found that Great Plains was negligent in its supervision of Dewey. The SEC might take all of these actions against Great Plains Securities except A) arrest. B) issue a fine. C) censure. D) suspension.

A broker-dealer is a business. You can not really arrest a business. You can censure, suspend, or fine a business. Note that the SEC cannot arrest a natural person either (like Dewey Cheatham), but they can contact the Justice Department. LO 10.a A

A broker-dealer's business model allows for only the purchase and sale of securities for retail customer accounts. It does not execute, settle, or clear its customer's transactions, nor does it tend to any back-office functions such as sending trade confirmations or forwarding proxies. This broker-dealer would best be described as what type of firm? A) Full service B) Market making C) Introducing/fully disclosed D) Clearing agent/carrying agent

A fully disclosed introducing broker-dealer is what the word implies—it introduces its customer's business to a clearing firm. Clearing firms (often called carrying firms or agents) hold funds and securities and settle transactions (clear and process) for their correspondent introducing firms. Essentially, the clearing firm acts as the introducing firm's back office. LO 1.e C

The investment return of a variable annuity comes from A) computing the excess of the premiums received over the mortality experience. B) the performance of the selected subaccounts within a separate account. C) the assumed rate stated in the policy documents. D) the insurance company's general account.

A key feature of the variable annuity is that most of the premium is invested into the insurance company's separate account rather than the general account. Within the separate account, a number of subaccounts may be selected, depending on the investor's objectives. It is the performance of these subaccounts that provides the annuity's investment return. LO 4.f B

All of the following are possible actions of an investor who has received stock rights except A) hold the rights for a possible long-term capital gain. B) exercise the rights to purchase the new stock at a discount. C) allow the rights to expire unexercised. D) sell the rights for a short-term capital gain or loss.

A long-term capital gain would require a holding period of more than one year. Rights expire four-six weeks after issue, so this would not be possible. LO 2.g A

A customer has multiple accounts at a bank that also owns accounts at a broker-dealer. All of the following accounts would be covered by FDIC insurance up to the specified limits except A) both spouses' individual savings account. B) a joint savings account with a spouse. C) an IRA in a five-year CD. D) a checking account in a money market mutual fund.

A money market mutual fund is not considered a deposit of the bank; it is a security, so it is not covered by the FDIC. LO 10.c D

Which of the following is a money market security? A) A 30-year T-bond issued by the Treasury 29 years ago B) A TAN maturing in 14 months C) A newly issued T-note D) A short-term T-bond mutual fund

A money market security is a high quality and highly liquid security with one year, or less, left to maturity. Both the T-note and the Tax Anticipation Note are more than a year from maturity. The mutual fund has no maturity. LO 3.f A

Being secured by no physical asset and backed only by a bank's good faith and credit, a bank's promise to pay principal and interest can be evidenced in which of the following securities that are traded in the secondary market? A) CDs B) Commercial paper C) Notes and bonds D) Negotiable CDs

A negotiable CD is an unsecured money market instrument issued by banks. Negotiable means that it can be traded in the secondary market and unsecured means that it is backed only by a promise to pay—a bank's good faith and credit. LO 3.f D

Passive investments do not fall under the definition of an outside business activity (OBA). Which of the following investments would be considered a passive investment? A) A partnership interest in a real estate brokerage firm B) A partnership interest in a local credit union C) A partial ownership in a car dealership D) Limited partnership unit

A passive investment, such as the purchase of a limited partnership unit, is not considered an outside business activity (OBA). An associated person may make a passive investment for his own account without providing written notice to the employing broker-dealer. LO 12.e D

Which of these would be unlawful regarding the use of a mutual fund prospectus? A) Failing to highlight a small section the customer has specifically asked about B) Calling an investor's attention to a section that may be interesting C) Leaving a typographical error in the text unmarked D) Sending a prospectus to someone who has shown no interest in the fund

A prospectus for any security, not just for a mutual fund, may not be marked; highlighted; or otherwise altered in any way; nor may steps be taken to call an investor's attention to some passage or section that might be of special interest, even if the potential customer asked that it be done. LO 4.e B

An investor is short 1 December 15 put at 6. The investor's maximum loss on this position is A) $60. B) $2,100. C) $1,500. D) $900.

A put writer's maximum loss is the put's strike price (15) less the premium received (6)—in this case, 9 points. Note that this is the same as the breakeven. This maximum loss occurs when the stock price drops to zero. The investor is forced to buy the worthless stock at the option's strike price of 15 and, therefore, has lost 15 points. The investor's total loss (15), however, is reduced by the premium (6) received, making the ML 9 points ($900). LO 5.a D

When must a report listing all written complaints be filed with FINRA? A) Within 30 days of the end of each calendar quarter B) Within 15 days of the end of the calendar year C) Within 15 days of the end of each calendar quarter D) Within 30 days of the end of the calendar year

A report listing all written complaints is filed with FINRA within 15 days of the end of each calendar quarter. Copies of complaints must be held for four years after resolution. LO 12.c C

Sam Malloy owns a small business and has built a substantial estate both with his business success and his early career as a pro athlete. He wants to set up his estate in a way that he will control the assets until he passes away or becomes incapacitated. Once that time comes, he wants control to transfer easily and he wants to avoid probate. Sam should A) create a last will and testament. B) establish an irrevocable living trust. C) establish a revocable living trust. D) place his assets in a transfer on death account.

A revocable living trust will accomplish his goals for his estate. An irrevocable trust takes away his control of his assets. The business cannot be placed in a transfer on death account. A will must go through probate. LO 6.c C

Which of the following is not correct regarding a rights offering? A) The rights offering allows the holder to exercise and purchase the stock at a price lower than the market. B) The subscription period is typically 30 to 45 days. C) The exercise period is typically long term, five years or more. D) Rights are issued to existing shareholders on a one-right-for-one-existing-share basis.

A rights offering allows stockholders to purchase common stock below the current market price. The rights are valued separately from the stock and trade in the secondary market during the subscription period, which is typically 30 to 45 days. Existing shareholders receive one right per share owned. The number of rights required to purchase one share of the new issue depends on the number of outstanding shares and the number of new shares offered. LO 2.g C

A risk that is specific to a particular issue or issuer A) impacts a broad group of securities equally. B) cannot be reduced by diversification. C) is a nonsystematic risk. D) is a systematic risk.

A risk that is specific to a particular issue or issuer is part of the definition of nonsystematic risk. It may be mitigated by diversification. LO 7.a C

Lando Entertainment, Inc., issues a bond collateralized by a trust holding the company's Las Vegas headquarters. This type of bond is called a A) mortgage bond. B) collateral trust bond. C) guaranteed bond. D) headquarters debenture.

A secured bond backed by real estate is called a mortgage bond. Collateral trust bonds hold other securities in trust as collateral. A guaranteed bond is an unsecured bond backed by a third party. A headquarters debenture is a fictional thing. LO 3.c A

Which of these broker-dealers would most likely have correspondent firms? A) A self-clearing firm B) An introducing broker-dealer C) A fully disclosed broker-dealer D) A market maker

A self-clearing (or carrying) firm holds funds and securities of the fully-disclosed or introducing firm's customers and performs related functions, such as sending confirmations and statements for them. Those firms, for whom the carrying firm performs those services, are known as its correspondents. LO 1.e A

A client entering a sell limit order at 43 would accept which of these trades? A) 44 B) 42.90 C) 42 D) 42.50

A sell limit at 43 means the investor will only accept a price of 43 or better (higher). Certainly, if the client is willing to sell at 43, she would be even happier to receive 44. LO 1.g A

A serial bond is best described as A) debt structured so that the principal of the whole issue matures at one time. B) portions of bond principal scheduled to mature at intervals over a period of years until the entire balance has been repaid. C) bonds in which the principal is secured by food-quality grains. D) the issuer repaying part of the bond's principal before the final maturity date, but paying off the largest portion of the bond at maturity.

A serial bond issue schedules portions of the principal to mature at intervals over a period of years until the entire balance has been repaid. LO 3.a B

When an issuer schedules portions of a bond issue's principal to mature at predetermined intervals over a period of years until the entire balance has been repaid, the issuer has issued what type of bond? A) Predetermined B) Balloon C) Term D) Serial

A serial bond issue schedules portions of the principal to mature at intervals over a period of years until the entire principal balance has been repaid. LO 3.a D

A traditional pension plan is an example of which of the following? A) A defined contribution plan B) A simplified employee pension C) A non-qualified plan D) A defined benefit plan

A traditional pension plan is a defined benefit plan. All defined benefit plan are qualified plans. LO 6.f D

Your client, Ann Porter, likes fast cars and has been saving for a high-end Italian sports car. She recently saw a report that said the dollar was likely to drop in the near future. She is concerned that this might affect her plans to buy her dream car next year. You tell her A) she should not waste her money on a fancy car. B) no, it should have no impact on her plans at all. C) yes, it will likely cost her less if the dollar drops. D) yes, it will likely cost her more to buy the car if the dollar drops.

A weakening dollar will likely cause the cost of her foreign made car to increase in dollar terms. LO 9.f D

Brokers placing orders for their own account ahead of notably large customer orders that are known to be entering the market in an attempt to gain from the price movement that is likely to occur is an example of what prohibited activity? A) Front running B) Marking the close C) Insider trading D) Breakpoint sale

Although each of these choices is a violation of Financial Industry Regulatory Authority (FINRA) rules and securities laws, when an investment professional places an order for his own account or one that he controls ahead of other orders that are so large that they will likely move the market in an attempt to gain from the price movement is an illegal activity known as front running. Those caught front running would be subject to severe sanctions by market regulators. LO 11.c A

A customer is not required to sign which of the following when opening a margin account? A) Credit agreement B) Margin agreement C) Hypothecation agreement D) Consent to loan form

Although most broker-dealers will not open the account without having the customer sign all of these, there is no rule that requires the customer to sign the consent to loan form. LO 6.g D

All of the following information is required to be provided on the Form U4 except A) disclosure of criminal charges. B) 5-year residency history. C) education (degrees or designations). D) 10-year employment history (including full- time education).

Although there is a place to disclose certain earned professional designations such as CPA and CFA, there is no requirement to disclose education (degrees or designations). Time spent as a full-time student however would be included in the 10-year employment history. LO 12.a C

The common stock of a U.S. corporation and an American depositary receipt (ADR) issued in the United States share all of the following types of risk except A) market risk. B) business risk. C) currency risk. D) regulatory risk.

An ADR represents a foreign stock and is subject to currency risk, even if the ADR is issued in the U.S. Like all equities, it is subject to market and business risk and is certainly exposed to regulatory risk in its home country. LO 2.e C

For initial public offerings (IPOs) of common stock, all of the following would be considered restricted persons except A) a person owning at least 5% of the member firm. B) fiduciaries acting on behalf of the underwriters. C) a member firm. D) employees of the member firm.

An individual person or entity would have to own 10% or more of a member firm before they would be considered a restricted person. LO 1.a A

Sam Malloy owns a small business and has built a substantial estate both with his business success and his early career as a pro athlete. He would like to begin to move assets out of his estate in a way that will allow him to benefit from the assets, but also allows for an easy transfer to his heirs when he dies. He needs to lower the size of his estate before he passes and hates the idea of a public hearing that is part of probate. Sam should A) place his assets in a transfer on death account. B) create a last will and testament. C) establish an irrevocable living trust. D) establish a revocable living trust.

An irrevocable living trust will accomplish his goals. The assets in a revocable trust remain part of his estate as do the assets in a TOD account and a will. Also, a will must be probated. LO 6.c C

Which of the following issues only common stock? A) A face-amount certificate company B) An equity unit investment trust C) A closed-end management investment company D) An open-end management investment company

An open-end (mutual fund) management investment company may only issue redeemable common stock. A unit investment trust offers units of beneficial ownership. A closed-end management investment company may also issue bonds and preferred stock, while a face-amount certificate company offers a contract, as opposed to units or shares. LO 4.a D

A trade of an equity index option settles when? A) The trade date B) The day after the trade C) The next business day D) Two business days

An option trade settles the next business day (as opposed to the broader "next day"). LO 5.b C

A 65-year-old investor is looking to earn additional income in her securities portfolio without much risk. Which of the following would likely be the least acceptable trading strategy for this investor? A) Purchasing debentures B) Covered call writing C) Purchasing junior preferred stock D) Naked call writing

An uncovered (naked) writer does not own the underlying security. If a call option is exercised by the owner, the writer must purchase the underlying security at the current market price to deliver it. There is a theoretical unlimited upside risk to this and is wholly unsuitable for a risk averse-investor. If the call contract is covered, the writer owns the underlying security. This ensures the writer's ability to deliver should the contract be exercised by the owner posing no upside risk to the investor from the sale of the option. LO 5.a D

A customer wishes to deposit $20,000 in cash into her account. This requires A) principal approval. B) written verification of the source of funds. C) filing a CTR with FinCEN. D) filing a SAR with FinCEN.

Any cash transaction of over $10,000 in cash must be report to FinCEN using the Currency Transaction Report. Reps must be aware of their firm's procedures, however, because many firms have more stringent internal requirements (such as receiving principal approval). LO 11.b C

Regarding the purchase of new equity issues by restricted persons, which statements are true? I. An investment club is permitted to buy a new equity issue at the offering price. II. An investment club is not permitted to buy a new equity issue at the offering price. III. An investment club that has eight members with equal ownership, one of which is a registered representative, is permitted to buy a new equity issue at the offering price. IV. An investment club that has 12 members with equal ownership, one of which is a registered representative, is permitted to buy a new equity issue at the offering price. A) II and III B) I and IV C) I and III D) II and IV

As long as an investment club has no restricted persons as members, it may purchase new equity issues at the public offering price. An investment club that has restricted persons as members may still participate in an initial public offering (IPO) so long as the total ownership of the club's assets by restricted persons does not exceed 10%. A registered representative is a restricted person under the rules regarding the purchase of new equity issues. In III the registered representative owns 12 ½ % (100% ÷ 8 = 12 ½) of club's assets. In IV the registered representative owns 8 1/3% (100% ÷ 12 = 8 1/3), under the 10% maximum allowed. LO 1.a B

A municipal finance professional (MFP) is A) an elected official of a municipality having some decision-making authority regarding who will underwrite the municipality's bonds. B) employed by a municipality (not elected) to oversee the issuance of municipal bonds. C) an employee of a Financial Industry Regulatory Authority (FINRA) member engaged in municipal security representative activities such as underwriting and trading. D) an employee of the Municipal Securities Rule Board (MSRB) specializing in seeing that broker-dealers adhere to the MSRB rules and regulations regarding the sales of municipal bonds.

As per the Municipal Securities Rulemaking Board (MSRB), a municipal finance professional (MFP) is an associated person of a member firm who is primarily engaged in municipal securities representative activities, including underwriting, sales and trading, or any other activity that involves communications with the public regarding municipals. LO 12.f C

You should expect which of these to occur when the dollar strengthens against other currencies? I. Imports will become less expensive. II. Inflation will go down. III. Inflation will rise. IV. Imports will become more expensive. A) I and II B) I and IV C) II and IV D) I and III

As the dollar gains strength against other currencies, the cost of imports goes down in dollar terms. Domestic producers will need to compete with the less expensive imports. That keeps prices overall from rising, reducing inflationary pressures. LO 9.f A

Representative Pete received a call from his client, Neil, to place a trade. He wanted to buy 200 shares of the Starshine Entertainment Company. Pete asked Neil a few questions about the trade before placing it. This is A) an unsolicited trade. B) a solicited trade. C) an unclassified trade. D) a discretionary trade.

As the representative did not introduce the trade to the customer, this is an unsolicited trade. The customer provided the three key elements of the order (Action, Asset, Amount), so it is not a discretionary trade. LO 6.h A

Who must grant permission for a representative to have discretionary trading authority on an account? A) The customer, the principal, and the representative B) The customer and the representative C) The customer and a principal of the broker-dealer D) The representative and a principal

Authorization for discretionary trading must come from both the customer and the firm (represented by a principal). The representative's authorization is not a requirement. LO 6.h C

BigCo corporation common stock currently trades at $30 per share and pays a $0.30 dividend. What is BigCo's dividend yield? A) 3% B) 4% C) 1% D) 2%

BigCo pays a $0.30 quarterly dividend. Dividend yield is an annual figure. To calculate the current dividend yield, you would multiply the quarterly dividend by four, and then divide the result by the current market value. (0.30 × 4)/30 = 1.2/30 = .04 (4%) LO 8.a B

Caleb McCann got a tip from his brother Nate on XYZ stock two months ago. Caleb hasn't previously been investing in the stock market but has been watching this stock since he got the tip from his brother. Caleb is now very bullish on XYZ and wants to place a trade. Which of the following would you recommend? A) Sell XYZ to close a long position B) Buy XYZ to open a long position C) Buy XYZ to close out a short position D) Sell XYZ to open a short position

Because McCann has no existing position, this would be an opening transaction, and if bullish on the stock it, would be a purchase. LO 1.g B

Which of these reasons would allow for a municipality to issue revenue bonds easier instead of general obligation bonds? I. Revenue bonds do not require voter approval. II. Revenue bonds generally have a higher rating than GO bonds from the same issuer. III. Revenue bonds are not constrained by a statutory debt limit. IV. Revenue bonds are supported by ad valorem taxes. A) I and III B) II and IV C) I and IV D) II and III

Because revenue bonds are designed to be self-supporting from the revenue derived from the project funded by the bonds, voter approval is not required. On the other hand, because GO bonds are backed by taxes, such as ad valorem taxes, voter approval is generally required and there is a debt ceiling or limit imposed on the issuer. LO 3.d A

While preferred shares tend to be less volatile than common shares, one type of preferred is noted as being even more stable in price than the others. This would be A) convertible. B) callable. C) participating. D) adjustable rate.

Because the dividend payment adjusts to current interest rates, the price of the stock remains relatively stable. In other words, it is the return that fluctuates rather than the price. LO 2.d D

In regards to fiscal policy, which of these statements is correct? I. Fiscal policy is considered the most efficient means to solve short-term economic problems. II. Fiscal policy is not considered the most efficient means to solve short-term economic problems. III. Fiscal policy refers to governmental budget decisions enacted by the U.S. president and Congress. IV. Fiscal policy refers to governmental budget decisions enacted by the U.S. president and the cabinet. A) II and IV B) I and III C) I and IV D) II and III

Because the political process determines fiscal policy, it takes time for conditions and solutions to be identified and implemented. Therefore, it is not considered an efficient way to solve short-term economic problems. Fiscal policy is the responsibility of the U.S. president and the Congress. LO 9.e D

Two weeks ago, Representative Pete introduced his customer Neil to the Windmill Growth Fund in response to Neil's interest in growth funds. Today the customer called Pete to place a trade to invest $10,000 in the Windmill Growth Fund. This is A) a discretionary trade. B) an unsolicited trade. C) a solicited trade. D) an unclassified trade.

Because the representative introduced the security to the customer, this is a solicited trade. The customer provided the three key elements of the order (action, asset, amount); it is not a discretionary trade. LO 6.h C

The buyer of a put has A) the right to buy the stock. B) the obligation to sell the stock. C) the right to sell the stock. D) the obligation to buy the stock.

The buyer of a put has the right to sell the stock. The seller of the put has the obligation to buy the stock. LO 5.a C

A company that is extensively overleveraged using debt financing whenever available would be exposing its investors to A) business risk. B) financial risk. C) liquidity risk. D) call risk.

Debt financing or utilizing debt leverage too much can lead to the inability to meet principal and interest payments on a company's debt obligations. This is the definition of financial risk. LO 7.a B

Which of the following is not a measure of a company's short-term liquidity? A) Working capital B) Debt ratio C) Current ratio D) Acid test ratio

Debt ratio is the measure of a company's long-term debt as a percentage of net worth. The others are derived from current asset and current liabilities, focusing on a company's short-term liquidity and cash demands. LO 9.d B

Producers would generally report rising inventories during which period of the business cycle? A) Expansion B) Recovery C) Peak D) Contraction

Downturns in the business cycle (a contraction) tend to be characterized by rising inventories (a sign of slackening consumer demand). During expansion (recovery is a synonym) and peak, goods are usually moving as fast as they can be produced with little chance of buildup in inventories. LO 9.a D

GHI Corporation reported the following financial results for the prior quarter: Earnings: $50 million Interest: $5 million Taxes: $15 million Depreciation: $1 million Amortization: $1 million What is GHI's EBITDA? A) $30 million B) $45 million C) $28 million D) $50 million

EBITDA is an acronym referring to earnings before interest, taxes, depreciation, and amortization. LO 9.d D

Seabird Coffee Company has just reported an unexpected increase in EBITDA this past quarter. This means they experience an increase in A) earnings available to common shareholders. B) inventory turnover. C) employee benefit costs. D) earnings before interest, taxes, depreciation, and amortization.

EBITDA is an acronym referring to earnings before interest, taxes, depreciation, and amortization. LO 9.d D

An amended Form U5 must be filed and a copy sent to the former employee within how many days of discovery of the inaccuracy? A) 30 B) 25 C) 90 D) 10

If a broker-dealer discovers that a filed Form U5 was inaccurate, an amended form must be filed and sent to the former employee within 30 days of the discovery of the inaccuracy. LO 12.a A

Which of the following characteristics are typical of an exchange-traded product (ETP)? A) The value of an ETP is derived by formula disclosed in the prospectus and it trades on an exchange only after normal trading hours. B) An ETP is an ineligible investment for retail customers, and may not be purchased on margin. C) The value of an ETP is derived from other investment instruments, and it trades on a national securities exchange. D) An ETP is marginable but may not be sold short.

ETPs are priced so that the value of the product is derived from other investment instruments, such as a commodity, a currency, a share price or an interest rate. ETPs are benchmarked to stocks, commodities or indices. They are marginable and may be sold short. LO 5.j C

Earned income would include all the following except A) tips. B) commission on sales for a real estate agent. C) year-end bonuses. D) dividends earned on mutual funds.

Earned income includes wages, salary, tips, bonuses, and income from active participation in a trade or business. LO 8.d D

Which of the following would be considered earned income? A) Interest received from a bond investment B) Bonus received from employment C) The premium kept from an unexercised short put D) Dividends received from a stock investment

Earned income is received as the result of participating in trade or business, the generation and/or sale of goods and/or services—in other words, from work. The other choices are earnings from investments and are known as portfolio income. LO 8.d B

The best characterization of how economists view the money supply is A) all forms of cash and liquid instruments but no forms of credit. B) savings and checking accounts and all lines of credit but not paper money or coins. C) cash, loans, different forms of credit, and other liquid instruments. D) paper money and coinage only.

Economists take a broad view of the money supply and include within it all cash (paper money and coins), loans, credit, and other liquid instruments, such as savings and checking accounts. LO 9.h C

Which of these is correct regarding the ex-date for a common stock? It is set by the board of directors. It is set by FINRA or the exchange. It is the first date an investor can purchase a security and not be entitled to the dividend. It is the date the seller reimburses the buyer for the amount of the dividend paid. A) II and IV B) II and III C) I and III D) I and IV

Ex-dates, for securities that trade in the secondary markets, are set by the market center where the trade occurs. It represents the day the new owner of a security will no longer receive the dividend if the trade settles regular way. LO 4.d B

Jackson Raleigh, a registered representative in Memphis, TN, has a client who is a pension fund manager for the Tiger Pension Fund. Raleigh creates several flyers of informational literature about the funds available in the Tiger Pension Fund and emails them to the fund manager so that the fund manager can prints copies of the flyers and make them available to the participants in the pension fund. FINRA would classify these flyers as A) retail communications. B) correspondence. C) institutional communications. D) sales literature.

FINRA has three classifications of communication with the public. Correspondence is communication to 25 or fewer retail investors in a 30-day period. Retail communications is to more than 25 retail investors in a 30-day period. Institutional communication is going to banks, insurance companies, mutual funds, et cetera. Even though the material was sent to the pension fund, which could be considered institutional communications, because the material was being forwarded to retail investors, it is considered retail communication. LO 11.i A

Caleb Wilson receives an email from his client, Shelby Bogdin, requesting the year-to-date performance on several of her mutual funds. Wilson immediately looks up the performance and emails her a response. This type of communication would be classified as under FINRA rules as A) respondence. B) retail communication. C) prohibited. D) correspondence.

FINRA has three classifications of communication with the public. Correspondence is communication to 25 or fewer retail investors in a 30-day period. Retail communications is to more than 25 retail investors in a 30-day period. Institutional communication is going to banks, insurance companies, mutual funds, et cetera. Responding to an email is not prohibited as long as the rep follows his company policies. Respondence is a word, however, it has nothing to do with FINRA communication rules. LO 11.i D

FINRA is the primary regulator in the United States for I. broker-dealers II. registered representatives III bankers IV. the SEC A) II and III B) I and II C) I and III D) I and IV

FINRA oversees broker-dealers and their associated persons, like registered representatives. FINRA's authority is delegated from the SEC, and they do not regulate banks, though they do regulate broker-dealers that are owned by banks. Investment banking activities performed by broker-dealers do fall under FINRA's jurisdiction. LO 10.e B

A 73-year old client in the 25% income tax bracket withdraws $20,000 from her traditional IRA. Based on her life expectancy, the withdrawal should have been $30,000. How much tax will she owe? A) $10,000 B) $5,000 C) $12,500 D) $7,500

Failure to meet the required minimum distribution results in a 50% penalty on the shortfall. In this case, she took $20,000 when she should have taken $30,000 so there will be a 50% tax on the $10,000 difference ($5,000 penalty). In addition to that $5,000 penalty, the ordinary income tax on the total amount that should have been withdrawn must also be paid (25% × $30,000 = $7,500). Total tax liability on the withdrawal equals $12,500 ($5,000 penalty tax plus $7,500 ordinary income tax). LO 6.e C

Federal Reserve member banks needing to borrow money can borrow from A) the Federal Reserve Bank at the federal funds rate. B) the Federal Reserve Bank at the discount rate. C) nonmember banks at the federal funds rate. D) member firms at the discount rate.

Federal Reserve member banks needing to borrow have two resources: the Federal Reserve Bank itself, which will lend to them at the discount rate, and other member banks, who will lend to one another at the federal funds rate. LO 9.h B

Regarding the potential financial exploitation of seniors, impacted accounts would be those for individuals A) who at any age are unable to protect her own interests. B) age 55 and older, or age 21 and older who the member reasonably believes has a mental or physical impairment that renders the individual unable to protect her own interests. C) age 65 and older, or age 18 and older who the member reasonably believes has a mental or physical impairment that renders the individual unable to protect her own interests. D) who are past the age of 75 and the member reasonably believes has a mental or physical impairment that renders the individual unable to protect her own interests.

Financial Industry Regulatory Authority (FINRA) has specifically identified for the purpose of preventing financial exploitation of seniors individuals who are age 65 and older, or age 18 and older who the member reasonably believes has a mental or physical impairment that renders the individual unable to protect her own interests. LO 11.f C

Registered representatives must complete or satisfy each of the following except A) the regulatory element. B) the Form U5. C) the firm element. D) state registration requirements.

Financial Industry Regulatory Authority (FINRA) requires that registered representatives complete both the regulatory and the firm elements of a continuing education (CE) program. In addition to satisfying FINRA requirements for registration, each state has its own requirements that must be satisfied before a representative can act in that state. A representative is not required to file a Form U5. That form is filed by the member firm upon the termination of a registered representative for any reason. A copy will be provided to the departed representative. LO 12.a B

Which of the following is a financial consideration? A) Is their child interested in college? B) How concerned are they about carrying debt? C) Where would they like to retire? D) Monthly interest costs of consumer debt loans.

Financial considerations are those issues that may be defined by a dollar amount (i.e., either a cash flow [$X per month] or a lump sum [$X once]). If the answer isn't a dollar amount, it is probably nonfinancial. LO 7.d D

An AML compliance officer should do all of the following except A) monitor compliance with all BSA requirements at the firm. B) notify customers immediately when an SAR is being filed. C) insure the firm is meeting all requirements of the Bank Secrecy Act for opening new accounts. D) insure the firm is properly training employees on the requirements of the BSA.

Firms are prohibited from notifying customers when an SAR is filed. LO 11.b B

An exchange has some degree of regulatory authority over all of the following except A) floor traders. B) dealers in the third market. C) designated market makers. D) exchange members.

Firms that make a market, in listed stocks in the OTC market, are under FINRA oversight. Those who do transactions on the floor are under the exchanges regulations. LO 10.f B

To grow or expand the economy, U.S. fiscal policy should be to A) raise taxes and cut all government spending for programs and development. B) cut taxes and increase government spending for programs and development. C) cut taxes and government spending for programs and development. D) raise taxes and government spending for programs and development.

Fiscal policies to grow or expand the economy would encompass cuts in taxes allowing consumers to have more money to spend, spurring the economy forward, and increasing government spending for programs and development that creates jobs, again spurring the economy forward. LO 9.e B

Fiscal policy seeks to encourage or discourage economic activity through the A) management of money supply and taxation. B) use of government spending and interest rate controls. C) use of government spending and taxation. D) management of money supply and government spending.

Fiscal policy is the use of government spending and taxation to smooth out the business cycle. Interest rates and money supply are elements of monetary policy. LO 9.e C

Which of the following are considered tools used to implement fiscal policies? I. Government spending II. Operations of the Federal Open Market Committee (FOMC) III. Changing the reserve requirements IV. Taxation A) II and III B) I and IV C) I and III D) II and IV

Fiscal policy refers to governmental budget decisions enacted by the president and Congress to regulate federal spending and taxation. Increases in taxes and decreases in government spending slow the economy, while lowering taxes and increasing government spending spur growth in the economy. LO 9.e B

Which of the following is an example of a company in a defensive industry? A) Pan American Aircraft, an airplane producer B) National Motors, an automobile manufacturer C) Major Mills Grain Company, a grain and cereal producer D) Centipede Equipment, a heavy equipment manufacturer

Food manufacturers are examples of companies that maintain sales even in down-cycles; defensive industries. The others all represent industries whose sales fluctuate with economic activity; cyclical industries. LO 9.c C

If a callable bond is priced at par, which of the following is true? A) Yield to maturity (YTM) is less than yield to call (YTC). B) Current yield (CY) equals yield to call (YTC). C) Current yield (CY) is less than yield to maturity (YTM). D) Current yield (CY) is greater than yield to maturity (YTM).

For any bond priced at par, all of the yields are equal; nominal = CY = YTM = YTC if callable. LO 3.a B

A common stock shareholder's residual right to corporate assets refers to which of the following? A) During the dissolution of corporate assets, common shareholders will be paid if any funds are left after debtholders are paid but before preferred shareholders are paid. B) During the dissolution of corporate assets, common shareholders will be paid if any funds are left after preferred shareholders are paid but before debtholders are paid. C) During the dissolution of corporate assets, common shareholders will be paid first— before debtholders and preferred shareholders are paid. D) During the dissolution of corporate assets, common shareholders will be paid if there are any funds left after debtholders and preferred shareholders are paid.

For common shareholders, having a residual right to corporate assets means that they will only be paid in the event of a corporate dissolution if there are any funds left after debtholders and preferred shareholders are paid. LO 2.c D

If a call contract has no intrinsic value, it must be A) at, or in the money. B) at, or out of the money. C) out of, or in the money. D) only in the money.

If a contract has no (zero) intrinsic value, it can only be either at or out of the money. Anytime a contract has intrinsic value, it is in the money by that amount. LO 5.a B

Which of the following is an investment recommendation? A) A representative cold-calls a potential customer and asks to set up an appointment. B) In response to a customer's questions about the Windmill Bond Fund, a representative provides information about the fund. C) During a presentation, a representative uses a chart of the S&P 500 to demonstrate the history of the stock market. D) In response to a customer's request about bond investments, a representative provides information on the Windmill Bond Fund.

If a representative introduces a specific product in response to a customer's generic question, it will be seen as a recommendation. Simply trying to schedule an appointment is not an investment recommendation. If a representative provides information on a specific investment that a customer has requested, it is not a recommendation. Basic information on the markets will not be considered a recommendation. LO 7.c D

When a registered person leaves the securities business, Financial Industry Regulatory Authority (FINRA) retains jurisdiction over that person for how long? A) 2 years B) 1 year C) In perpetuity D) 30 days

If a terminated person becomes subject to a customer complaint or charges are brought against that person by FINRA, that person remains subject to FINRA jurisdiction for two years following termination. LO 12.a A

If a customer sold puts to open, which of the following transactions would be allowed if the options agreement was not returned signed within 15 days? A) Buy puts to close B) Sell puts to open C) Buy calls to close D) Sell calls to open

If the agreement is not returned signed in 15 days, only closing transactions to offset those positions already open would be allowed. LO 5.c A

A client and his spouse own shares in the KAPCO Fund as tenants in common. He has a 60% ownership interest in the account and the spouse has the balance. If the client dies, what happens to the shares in the account? A) Ownership of the shares would be determined by probate court. B) 40% of the shares would belong to his spouse and the remaining balance would be distributed to his estate. C) His interest would automatically be transferred to the spouse. D) 50% of the shares would belong to his spouse and the remaining half would be distributed to his estate.

In a TIC account, securities owned by the decedent pass to the deceased owner's estate—in this case, 60% of the assets. The 40% belonging to the spouse is retained by the spouse. LO 6.a B

Marsha, Jane, Cynthia, Craig, Jim, and Robert are owners of an account JTWROS. If Craig, Jim, and Robert pass away, then their interest in the account A) is identified and distributed with the decedent's estate. B) is distributed through the probate process. C) remains in the account and is now the property of the surviving tenants. D) is divided in half, and one half of the account is distributed evenly to the decedent's beneficiaries.

In a joint tenants with rights of survivorship (JTWROS), the assets of the decedent simply remain in the account and become the property of the survivors. There is no probate process for these assets, but they are still a part of the decedent's estate for tax purposes. LO 6.a C

Which of the following are characteristics of a revocable living trust? I. It is established before the grantor dies II. The grantor can change beneficiaries III. The grantor can add or remove items from the trust IV. The grantor is not subject to tax on income that remains in the trust A) I, II, III, and IV B) I, II, and III C) I and II D) I only

In a revocable living trust the grantor has complete control over the trust while alive, and because of this, the grantor is also subject to any tax implications of the trust. LO 6.c B

A registered representative placing trades in a customer account must have discretionary authority if they choose which of the following aspects of the trade? A) The action to be taken and the asset to be traded B) The action to be taken, the asset to be traded, or the amount of the trade C) The action to be taken, the asset to be traded, and the amount of the trade D) The asset to be traded and the amount of the trade

In order for a trade to be considered discretionary the representative needs to choose any one or more of the three aspects of the trade (asset, action, or amount). It does not require more than one aspect, so the best response to the question is Action, Asset, or Amount. Any response that includes "and" suggests more than one of the "A"s needs to be controlled and is not accurate. LO 6.h B

An investor has a long position in ABC Chemical Corp. (ABCCC), with a substantial unrealized loss. Wishing to use that loss to offset realized gains, the investor sells the stock. In reinvesting the proceeds of the sale, the investor could avoid violating the wash-sale rule by purchasing A) ABCCC put options. B) ABCCC call options. C) ABCCC warrants. D) ABCCC convertible bonds.

In order to avoid violations of the wash-sale rule, investors selling a stock at a loss cannot purchase that same, or substantially identical, security within a 30-day period prior to or following the sale incurring the loss. Substantially identical would include anything that is exercisable or convertible into the same shares of stock; rights, warrants, call options, or a convertible bond. Note that when put options are exercised, the owner now has the right to sell the stock, not purchase it. Therefore buying puts in no way violates the wash-sale rule. LO 8.b A

In 2011, RST Corp. had both common stock and $100 par value 4% noncumulative preferred stock, outstanding. The preferred stock, like the common stock, pays dividends on a quarterly basis. Because of financial difficulties, the company stopped paying dividends after 2011. After resolving its problems in 2015, the company resumed dividend payments in 2016. Before paying the first quarterly common stock dividend that year, the company would have to pay a quarterly dividend to the preferred stockholders of A) $17. B) $20. C) $4. D) $1.

In the case of a noncumulative preferred stock, skipped dividends are forever lost. So, when the company is able to pay a dividend, which is always the case, it must pay the current preferred dividend prior to paying common. The question states that dividends are paid quarterly. Therefore, the quarterly dividend on a stock paying $4 annually would be $1—an amount that must be paid before the quarterly common dividend can be paid. LO 2.d D

When a company wants to issue additional shares of stock, the preemptive right given to existing shareholders allows those shareholders to A) maintain their proportionate ownership in the corporation. B) pass on their proportionate ownership in the corporation to an heir. C) decrease their proportionate ownership in the corporation. D) increase their proportionate ownership in the corporation.

In the event a corporation wants to issue additional shares of stock, the preemptive right given to existing shareholders allows the shareholders to maintain their proportionate ownership in the corporation by purchasing shares before the shares are available to new investors. LO 2.c A

Which of the following best fits the description of a growth stock? A) Common or preferred shares in companies, which experience growth in unusual, nonrecurring profitable circumstances B) Preferred shares in companies that back stated dividends with investments in pharmaceutical companies C) Common shares in companies that pay a high dividend on rapid growth experience D) Common shares in companies that retain earnings and pays little or no dividends

Most every industry passes through phases; introduction, growth, maturity, and decline. An industry is in its growth phase if it is growing faster than the economy as a whole due to e.g. technological changes, new products, or changing consumer tastes. Because growth companies retain nearly all of their earnings to finance business expansion, growth stocks pay little or no dividends. LO 9.c D

Which of the following securities trade at a price determined by formula? A) Common shares of a closed-end investment company B) Preferred shares of a corporation C) Common shares of a corporation D) Common shares of a mutual fund

Mutual funds are priced at the end of each business day, with sellers receiving the next calculated NAV and buyers paying the next calculated POP. All transition requests must be entered by 4:00 pm ET. Any requests to buy or sell that are entered after 4:00 pm will receive the next business day's NAV or POP. For example, a seller who places an order after the close (4:00 pm ET) on Friday will receive Monday's NAV when liquidating her shares. LO 4.a D

A mutual fund's public offering price is $10.50. An investor who wishes to invest $1,000.00 in the fund is able to purchase A) 95.238 shares. B) 95 shares with $2.50 left. C) only whole shares. D) 96 and owe $8.00.

Mutual funds may be purchased in even dollar amounts and partial shares may be issued. LO 4.d A

Which of the following is not part of the secondary markets? A) The exchanges B) Over-the-counter market C) Third market D) Mutual fund market

Mutual funds, as open-end investment companies, do not trade in the secondary markets. LO 1.d D

Negotiable jumbo CDs are characterized by all of the following except A) they trade in the secondary market. B) each issue generally matures in 5-10 years. C) they are unsecured debt of the issuing bank. D) they are issued in amounts of $100,000-$1 million.

Negotiable jumbo CDs are issued in denominations of $100,000-$1 million and trade in the secondary market. Most jumbo CDs are issued with maturities of one year or less. These CDs are unsecured promissory notes backed only by the credit standing of the issuing institution. LO 3.f B

ABC has the following recorded on its balance sheet: Current Assets$50,000Fixed Assets$100,000Notes payable$40,000Accounts Payable$25,000 ABC's net worth is A) $110,000. B) $85,000. C) $35,000. D) -$35,000.

Net worth is assets - liabilities. Therefore, ABC's net worth is 150,000 - 65,000 = 85,000. LO 9.d B

A new client of the member firm has just opened a margin account. After account approval, the client's initial trade is an order to purchase 100 shares of LMN common stock at $25. With Regulation T at 50%, in order to be in compliance with all regulations, the client would need to deposit A) $1,250. B) $1,000. C) $2,000. D) $2,500.

No borrowing can take place in a margin account without at least $2,000 in equity. It is only necessary to pay in full when the purchase is less than $2,000. It is only necessary to deposit more than $2,000 when the trade exceeds $4,000. LO 6.g C

Regarding preferred stock, which of the following is false? A) Preferred shareholders are paid before common shareholders in the event of a corporate bankruptcy. B) The right to vote on corporate issues is not available for preferred shareholders. C) Preferred shareholders have no preemptive rights that precede the preemptive rights of common shareholders. D) A corporation issuing common shares must issue at least one class of preferred shares.

No corporation must issue any class of preferred stock. Preferred shareholders have no voting rights or preemptive rights, but they are paid before common shareholders in the event of a corporate dissolution. LO 2.d D

A tombstone announcement may contain all of the following except A) an offer to sell the securities. B) type of security. C) names of the underwriters. D) number of shares offered.

No offer to sell can be made with a tombstone announcement. A tombstone is just information that an offer is coming to the market. LO 1.b A

All of these are true regarding no-load shares except A) they are sold by the fund with no sales charges or fees of any kind. B) they are redeemed with no charges or fees of any kind. C) they offer more return per dollar invested than load funds if investing results are the same. D) they have sales charges associated with sales and redemptions.

No-load shares have expenses that are not considered sales charges. Some broker-dealers may charge fees for transactions, but these fees are not from the fund. LO 4.b D

Which of the following is true regarding nonregistered personnel of FINRA member firms? A) Nonregistered employees of broker-dealers have an unlimited scope of activities as long as they are under the direct supervision of a registered representative. B) Nonregistered employees of broker-dealers have an unlimited scope of activities as long as they are under the direct supervision of a principal. C) Nonregistered employees of broker-dealers have a limited scope of activities. D) All employees of a member firm must be registered.

Nonregistered employees of broker-dealers have a limited scope of activities. They may not perform securities business even if supervised by registered persons. LO 11.h C

Systematic risk would include all of the following except A) market risk. B) interest rate risk. C) inflation risk. D) business risk.

Nonsystematic risks are those associated with the issuer (like a bad business strategy). Systematic risks impact large portions of the market and are difficult to reduce by diversification. LO 7.a D

When responding to regulatory requests for documents a broker-dealer A) must comply with the request within three months. B) need not comply unless they are convinced of good cause. C) may request a court hearing. D) must comply with the request promptly.

Prompt compliance with regulatory requests is required. By becoming a member, or registering with regulators, a broker-dealer agrees to provide documents and other evidence on request. LO 11.k D

A customer who has written an option contract receives an assignment notice. This customer is A) now obligated to short the stock at the current market value. B) the party who may now assign the contract to another investor. C) the party who must notify the Options Clearing Corporation (OCC) that they intend to exercise the contract. D) now obligated to either buy or sell the underlying stock at the strike price.

Only the owners of options contracts can exercise them. When this occurs, the writers of the contracts are assigned and must fulfill their obligation to perform. Performance (buy or sell) is determined by whether they have written a call (short call - obligated to sell) or a put (short put - obligated to buy). LO 5.c D

A business structure designed for an individual owner that allows for the results of the business to pass through to the owner is which of the following? A) S Corporation B) Partnership C) Sole proprietorship D) C Corporation

Partnerships and corporations are designed for multiple owners, and C corporations do not pass through. Sole proprietorships are designed for one individual owner, are easy to form, and allow the business results to pass through. LO 6.b C

What is the greatest disadvantage of limited partnerships? A) Lack of liquidity B) May invest in real estate C) May invest in oil and gas drilling D) The tax consequences

Partnerships are generally illiquid, and this drawback is one of the primary disadvantages with limited partnerships. LO 5.g A

A new employee has successfully completed the Securities Industry Essentials (SIE) exam. Which of the following activities is permissible for them to perform? A) Taking trades under the direct supervision of a principal B) Selecting literature for an appropriate investment for a customer C) Answering the phones and responding to noninvestment questions D) Taking down the details of a trade for a registered person to enter

Passing the SIE does not allow a person to become registered. A nonregistered person may not enter a trade or record the details to pass along to a registered person. Trade information must flow directly between a customer and a registered representative. How would a person determine what is an appropriate investment without performing a suitability analysis? LO 11.h C

The first stage of money laundering is A) integration. B) layering. C) placement. D) insertion.

Placement is the first of the three stages of money laundering, followed by layering and integration. Insertion is not one of the three steps. LO 11.a C

An investor purchased and then sold a security eight months later for a gain. This gain A) is considered to be a long-term gain, and it will be taxed at a more favorable rate than short-term gains. B) is considered to be a short-term gain, and it will be taxed at the same rate as the taxpayer's other ordinary income. C) is considered to be a short-term gain, and it will be taxed at a more favorable rate than long-term gains. D) is considered to be a long-term gain, and it will be taxed at the same rate as the taxpayer's other ordinary income.

Positions closed within 12 months or less are considered short term. When a gain is realized, it will be taxed at the same rate as the taxpayer's other ordinary income. By contrast, a long-term capital gain is taxed at a favorable long-term rate. LO 8.e B

Which of the following would likely be classified as a countercyclical industry? A) Auto manufacturing B) Home building C) Gold mining D) Food production

Precious metals and the companies that produce them are countercyclical (i.e., going up when the economy is shrinking, and down as the economy grows). LO 9.c C

A gold mining company is probably an example of what industry? A) Defensive B) Counter-cyclical C) Growth D) Cyclical

Precious metals are an example of a counter-cyclical industry. LO 9.c B

A customer of a broker-dealer sells 300 shares of stock at $50 per share and leaves the proceeds in the account. The proceeds are I. a free credit balance. II. a debit balance. III. available to the customer on demand. IV. available for withdrawal after 30 days. A) II and III B) II and IV C) I and IV D) I and III

Proceeds from a sale that are not reinvested and held in the account at the broker-dealer are considered a free credit balance. The free means that the funds are available to the customer on demand (freely available). LO 6.g D

Which of the following best describes the calculation for gains or losses for tax purposes? A) Proceeds minus cost basis B) Proceeds plus cost basis C) Proceeds minus dividend, plus cost basis D) Proceeds plus dividends, minus cost basis

Proceeds minus cost basis equals capital gains. The dividends are not part of the calculation for capital gains. LO 8.b A

Regarding capital gains, which of the following is true? A) Short-term gains are those realized on positions held for 12 months or less. B) Short-term gains are those realized on positions held for 9 months or less. C) Long-term gains are those realized on positions held for 2 years or more. D) Long-term gains are those realized on positions held for 10 years or more.

Profits on positions held 12 months or less are considered short-term gains. For those positions held longer than 12 months, the gains are considered long term and taxed at a more favorable rate. LO 8.e A

The Securities Investors Protection Corporation protects investors from the financial failure of A) the third market. B) broker-dealers. C) banks. D) the stock market.

The SIPC protects investors in the event of a collapse of their broker-dealer firm. LO 10.c B

The board of directors of the ABC Growth Fund has declared a $1-per-share dividend payable to holders of record on Wednesday, April 27. What is the most likely ex-dividend date for this dividend? A) Tuesday, April 26 B) Monday, April 25 C) Friday, April 29 D) Thursday, April 28

Purchasers of mutual fund shares become owners of record on the day the buy takes place. Sellers of mutual funds cease to be owners on the day the trade takes place. The result of the trade and settlement taking place on the same day is that you may buy the fund and receive the dividend as an owner of record on the same day. So when is the dividend no longer available to new owners? The day after the record date. The ex-dividend date for a mutual fund is the day after the record date. LO 4.d D

Which of the following is considered a systematic risk? A) Regulatory risk B) Purchasing power risk C) Default risk D) Financial risk

Purchasing power risk affects all fixed income securities to the same degree, making it a systematic risk. The others are all examples of nonsystematic risk. LO 7.a B

A customer's account has been frozen. Which of the following is true if the customer wants to purchase more securities? A) Approval must be received for any purchase in the account from the Securities and Exchange Commission (SEC).v B) Funds can be borrowed in the form of a margin loan up to 50% of the securities' value. C) No new purchases can be made in an account that has been frozen. D) Funds to pay in full must be available in the account before the buy order is entered.

Purchasing securities in a frozen account is permitted, but the funds to pay for the purchase in full must be available in the account before the order can be entered. LO 11.c D

Certain actions taken by the Federal Reserve Board (FRB) would likely have the effect of causing interest rates to increase. Which would these be? I. The Federal Open Market Committee (FOMC) buying securities II. Raising the reserve requirements III. Raising the discount rate IV. Raising the prime rate A) II and IV B) II and III C) I and IV D) I and III

Raising reserve requirements, having more member deposits being held on reserve at the Fed, would lessen the money available to lend. Raising the discount rate, charging member banks more for loans, would also lessen the money available to lend. With less money available to lend, interest rates would go up. LO 9.i B

Obtaining the financial status of the customer, and whether or not they meet income and net worth criteria, could be required for all of the following except A) real estate limited partnerships. B) equipment leasing limited partnerships. C) real estate investment trusts (REITs). D) oil and gas limited partnerships.

Real estate investment trusts (REITs) do not require proof of financial status for investment. Limited partnerships and other DPPs can, particularly those that are offered privately (as private placements) as opposed to those that are offered publicly (by public offering). LO 5.f C

After an extended period of high unemployment and negative GDP figures, the nation's most recent GDP is positive and employment is improving. This indicates the nation is entering a period of A) disaster. B) prosperity. C) recovery. D) contraction.

Recovery is the best answer. A recovery occurs when the economy begins to show some signs of growth after a period of contraction. The period of the trough (bottom) along with deflation and economic disaster are hopefully behind the nation. LO 9.a C

The nation has passed through very trying economic times. Many have lost jobs and are tapping into savings to maintain their lifestyles. The economy is not weakening and shows a few signs of growth, but the damage from the downturn is very visible. The nation is likely entering a period of A) deflation. B) trough. C) recovery. D) disaster.

Recovery is the best answer. A recovery occurs when the economy begins to show some signs of growth after a period of contraction. The period of the trough (bottom) along with deflation and economic disaster are hopefully behind the nation. LO 9.a C

Registered broker-dealers must have fingerprint records for most of their employees and for which of the following? A) Doorman B) Receptionist C) Partners D) Drivers

Registered broker-dealers must have fingerprint records made for most of their employees, and all directors, officers, and partners. Those who are ancillary to the securities business and do not speak about securities with the investing public or who do not handle funds or securities are generally not required to be fingerprinted. LO 12.a C

A broker-dealer's registration with the Securities and Exchange Commission (SEC) conveys that the SEC A) has made a positive determination regarding the financial strength of the member firm and has approved the member firm's business model. B) has approved the member firm's business model. C) has accepted the member firm's registration to do business. D) guarantees that the member firm will abide by all securities industry rules.

Registration with the SEC in no way implies that the SEC has passed upon or approved the broker-dealer's financial standing, business, or conduct. Any such claim or statement is misrepresentation. LO 10.a C

Your client, Teresa Jenson, calls and wants to purchase T-bills and wants to know when payment is due. You should tell her A) trade date. B) trade date plus 2 business days. C) trade date plus 1 business day. D) trade date plus 3 business days.

Regular way settlement is T + 2 for everything except treasuries and money market securities. Option trades and treasuries settle next business day (T + 1 ). LO 1.i C

Regular way settlement for common stock is A) trade date. B) trade date plus 3 business days. C) trade date plus 1 business day. D) trade date plus 2 business days.

Regular way settlement is T + 2 for everything except treasuries, money market securities, and options. LO 1.i D

A client buys stock on Monday, August 14, in a cash account. Under Regulation T, when is the client's payment due? A) In four business days B) In two business days C) At or before the time of order placement D) The same day

Regular-way firm-to-firm settlement is two business days after the trade date (T+2). Under Regulation T, payment must be made two business days after the settlement date (S+2 or T+4). LO 6.g A

Which of the following is not required to endorse a stock certificate for transfer? A) The correct number of shares are delivered B) Signatures of the receiving registered representative C) Certificates in good condition D) Proper endorsement by all owners

The broker-dealer does not sign the certificate. All the rest of these are requirements. LO 1.i B

ABC Corp stock is trading at BID 75.32, ask 75.35 5 x 12. What is the spread? A) $3.00 B) $0.35 C) $0.03 D) $0.30

The spread is the difference between the bid and ask. In this example it is three cents. LO 1.f C

Risk that prevails despite diversification within an asset class is A) systematic risk. B) business risk. C) call risk. D) unsystematic risk.

Risk that prevails despite diversification is one of the defining characteristics of systematic risk. LO 7.a A

An investor who is an affiliate of XYZ Corporation holds shares of restricted XYZ stock. These shares have A) no holding period but are subject to volume limits for as long as the investor is an affiliate. B) a six-month holding period and are subject to volume limits for as long as the investor is an affiliate. C) no holding period, nor are they subject to any volume limitations currently or in the future. D) a 30-day holding period and are subject to volume limits for as long as the investor is an affiliate.

Rule 144 mandates that restricted shares held by an affiliate are subject to a six-month holding period and will be subject to volume restrictions (limits) for as long as the individual continues as an affiliate of the corporation. LO 2.f B

Under the provisions of Rule 144, what percentage of outstanding stock may a control person sell every 90 days? A) 1% B) 4% C) 3% D) 6%

Rule 144 pertaining to the sale of restricted or control stock allows for the sale of 1% of the outstanding shares or the weekly average of the last four weeks' trading volume (whichever is greater), every 90 days. LO 2.f A

All of the following are restricted persons except A) employees of members. B) portfolio managers. C) finders and fiduciaries acting on behalf of the underwriters. D) individual owning 5% of a member firm.

Rules prohibit member firms from selling public offering stock in equities to any account in which restricted persons are beneficial owners. Restricted persons include Financial Industry Regulatory Authority (FINRA) members, employees of member firms, finders and fiduciaries acting on behalf of the underwriters, portfolio managers, and any person owning 10% or more of a member firm. Also included are the immediate family members of any restricted persons. LO 1.a D

Which of the following would require the filing of a SAR? Any transaction alone or in aggregate involving at least A) $5,000 on a single day. B) $,3000 on a single day. C) $3,000 and appears to serve no business or legal purpose. D) $5,000 and appears to serve no business or legal purpose.

SARs are required to be filed by the firm if the transaction appears to serve no business or legal purpose, and the transaction involves alone or in aggregate at least $5,000. LO 11.b D

Darrell has an individual account at Seacoast Securities, Inc., an SIPC firm that holds $300,000 in cash; $100,000 in various stock; and $100,000 in the Seacoast Money Market Fund. If Seacoast Securities fails how much coverage does Darrell have? A) $100,000 B) $250,000 C) $450,000 D) $350,000

SIPC covers securities and cash at a broker-dealer up to a maximum of $500,000, but no more than $250,000 in cash. A money market fund is a type of security. Darrell is covered for $200,000 in securities and $250,000 in cash for a total of $450,000. LO 10.c C

Which of the following are true regarding mutual fund sales charges? I. They are used to defray fund expenses, such as operating costs and salaries. II. They are set by the fund's transfer agent. III. They are not an expense to the fund, but to the investor. IV. They are used to compensate the fund's underwriter and sales representatives. A) I and IV B) III and IV C) II and III D) I and II

Sales charges are not among the standard business expenses of a mutual fund and may not be considered part of them. They are intended to compensate the underwriter and broker-dealers and their sales representatives who sell the fund shares to the public, and are thus expenses to the investor, not to the fund. LO 4.b B

Secondary markets exist to do all of these except A) allow individual investors easy access to investment vehicles. B) support the existence of primary markets. C) decrease liquidity in the national markets. D) allow investors to easily liquidate securities.

Secondary markets are focused on providing, not decreasing liquidity. Ultimately, a fair and orderly secondary market makes securities more attractive, supporting the functioning of the primary markets. LO 1.d C

Your customer, Shea, has a large portfolio of bonds and dividend paying stocks. Her primary interest is generating current income. She is trying to understand how taxes work for her T-bonds. You explain that A) the interest from her T-bonds is exempt at the state level, but she will still owe taxes at the local and federal level. B) the interest from her T-bonds is exempt at the state and local level, but she will still owe taxes at the federal level. C) the interest from her T-bonds is exempt at all levels. D) the interest from her T-bonds is exempt at the federal level, but she will still owe taxes at the state and local level.

Securities issued by the federal government produce interest that is not taxed at the state or local level. It is taxed at the federal level. LO 3.g B

To qualify as an exempt transaction under Tier 1 of Regulation A, an issuer may offer a maximum of which of the following? A) Up to $6 million in securities in a 12-month period B) Up to $75 million in securities in a 12-month period C) Up to $5 million in securities in a 12-month period D) Up to $20 million in securities in a 12-month period

Securities offerings up to $20 million in a 12-month period will be allowed in Tier 1. Of the $20 million, no more than $6 million can be sold on behalf of existing selling shareholders (similar to a combination offering). The offering would be subject to a coordinated review by individual states and the SEC. LO 1.c D

A bank employee has noticed that one of its customers has deposited $9,000 in his account every Monday, Wednesday and Friday for the past four weeks. Though the action could be legitimate, the customer might be trying to circumvent which of the following? A) The Trust Indenture Act B) The Maloney Act C) The Bank Secrecy Act D) The Securities Exchange Act

The Bank Secrecy Act requires, among other things, that transactions in currency amounting to more than $10,000 in a single day be reported on a Currency Transaction Report, CTR (Form 112). By keeping deposits under $10,000, the depositor might be engaging in one of the many forms of structuring. That is, structuring deposits in such a way so as to avoid the reporting requirements. LO 11.b C

During a discussion with a customer about a potential investment opportunity involving securities, standing alone, all of the following would likely be permissible except A) the registered representative shows the customer a brochure with a chart showing best and worst case scenarios for product development over the next year. B) the registered representative pessimistically implies that an investment has a good chance of losing money, as well as gaining, because the product might not have a market. C) the registered representative points out, correctly, that the maximum possible loss on this particular investment is probably smaller than the maximum possible gain. D) the registered representative points out only that a tech firm has a brilliant product idea and the CEO has advanced degrees in science.

The CEO may have advanced degrees, and the product idea may be brilliant, but the registered representative has failed to mention that these two things do not guarantee success nor the relevancy of the degree to the product or idea being discussed. LO 11.i D

Accusations of Financial Industry Regulatory Authority (FINRA) Conduct Rule violations will heard and handled under the A) Uniform Practice Code. B) Code of Procedure. C) Code of Arbitration Procedure. D) Uniform Securities Act.

The Code of Procedure describes how member violations of the Conduct Rules will be heard and handled. LO 10.e B

Any sale of securities outside an associated person's or the employing member firm's regular business is recognized as A) an unsolicited transaction. B) a nonissuer transaction. C) an outside business activity. D) a private securities transaction.

The Conduct Rules define a private securities transaction, also known as selling away, as any sale of securities outside an associated person's regular business and his employing member. LO 12.e D

The most common way of measuring purchasing power risk is A) the GNP. B) the GDP. C) the CPI. D) the DND.

The Consumer Price Index measures the increase or decrease in consumer prices. GDP and GNP are both measures of economic activity. DND is a fictional acronym, which has nothing to do with this test. Do not pick things you do not recognize just because it is unfamiliar to you. LO 9.b C

The Dow Jones Industrial Average is a price-weighted index that includes how many components? A) 30 B) 15 C) 20 D) 500

The DJIA covers 30 large-cap industrial companies. The Dow Jones averages are price-weighted, meaning that higher stock prices have more weight than lower stock prices. This point is not tested on the SIE. LO 7.e A

A customer has a significant amount of money in bank deposit accounts: $225,000 in a savings account titled in the customer's name; $240,000 in a checking account titled jointly with a spouse; and $100,000 in an account where the customer is custodian for a grandchild. Should that bank fail, the Federal Deposit Insurance Corporation (FDIC) insurance would cover A) $225,000 for the savings account, $100,000 for the custodial account, and nothing for the checking account. B) the entire $565,000. C) $250,000 for the savings and checking accounts and $100,000 for the custodial account. D) a total of $250,000, divided proportionately among the three accounts.

The FDIC provides deposit insurance guaranteeing the safety of a depositor's accounts in member banks up to $250,000 for each deposit ownership category in each insured bank. Each account listed (savings, checking, and custodial) is a separate ownership category under FDIC rules, so all the money in each of them is covered. LO 10.c B

Money available to lend to corporations and consumers is impacted most in the United States by the policies of which of the following? A) The National Securities Clearing Corporation (NSCC) B) The Internal Revenue Service (IRS) C) The Securities and Exchange Commission (SEC) D) The Federal Open Market Committee (FOMC)

The FOMC meets regularly to direct the Federal Reserve Board (FRB) to either buy or sell Treasury securities in the open market. Purchases add money to the economy, making the money available to lend more plentiful, and sales take money out of the economy, making money available to lend less plentiful. LO 9.h D

A margin account allows a customer to borrow a portion of the funds needed to complete a trade. Currently, the required minimum is 50%. Which regulator sets the requirement? A) The FRB B) FINRA C) The OCC D) The SEC

The Federal Reserve Board sets the Regulation T requirement. LO 6.g A

The monetarist theory proposes which of the following? A) The federal government can impact the economy by raising and lowering the federal funds rate. B) The Federal Reserve may impact the economy by raising and lowering the discount rate. C) The Federal Reserve has a major impact on the economy by raising and lowering taxes. D) The federal government impacts the economy through repurchase and reverse repurchase agreements.

The Federal Reserve controls the discount rate and repurchase and reverse repurchase agreements. The federal government controls taxes and spending. LO 9.h B

The Federal Reserve pursues its dual mandate through A) monetary policy. B) fiscal policy. C) trade policy. D) economic policy.

The Federal Reserve is tasked with using monetary policy to manage the economy. Fiscal policy—taxation and spending—is managed by the legislative and the executive branch. Trade policy is driven primarily by the executive branch, and all of this is part of economic policy. LO 9.g A

Which of these is not a division of the Treasury Department with some level of oversight for the securities industry? A) Office of the Comptroller of the Currency B) Financial Crimes Enforcement Network C) Financial Industry Regulatory Authority D) IRS

The Financial Industry Regulatory Authority (FINRA) is not a government agency, so it is not part of the Treasury Department. FINRA is a nongovernmental organization that has received significant delegated authority from the SEC. LO 10.b C

Craig and Judy have just married. It is a second marriage for both of them and they both have kids from a prior marriage. Craig would like his portion of their account to go to his kids when he dies and Judy would like her portion to go to her kids when she dies. As new partners in marriage, while they are both alive they would both like to have full access to the account. What type of account(s) should they set up? A) A partnership account B) Joint tenants with rights of survivorship (JTWROS) C) Joint tenants in common (JTIC) D) Each should set up their own individual transfer on death (TOD) account with limited power of attorney (POA)

The JTIC account does exactly what the clients requested. The JTWOS does not separate their assets at death; instead the whole account would go to the surviving spouse. Individual accounts with limited POA and TOD would not give both full access to the account. The partnership account is for business accounts. LO 6.a C

Which of these business structures would pass through the results of the business to the owners and protect the owners from the liabilities of the company? A) A C Corporation B) A sole proprietorship C) A general partnership D) An LLC

The LLC is the only one of these that both passes the income and losses through while providing liability protection. C Corps protect but do not pass through losses. General partnerships and sole proprietorships pass results through but offer no protection. LO 6.b D

MSRB rules apply to all of the following except A) municipalities. B) municipal dealers. C) investment bankers. D) FINRA member firms performing trades with municipal bonds.

The MSRB has no authority over municipal governments. They do make rules for underwriters (investment bankers) bringing municipal bonds to market, and municipal bond trading for both dealers and brokers acting on a customer's behalf. LO 10.f A

The MSRB has jurisdiction for making rules in all of the following cases except A) banks selling municipal securities. B) broker-dealers underwriting municipal securities. C) broker-dealers trading municipal securities. D) municipalities selling their own securities.

The MSRB makes regulations for broker-dealers and banks buying and selling municipal securities but does not have authority over the municipalities. LO 10.f D

Which of the following regulatory bodies regulates but has no enforcement powers? A) Financial Industry Regulatory Authority (FINRA) B) Chicago Board Options Exchange (CBOE) C) Securities and Exchange Commission (SEC) D) Municipal Securities Rule Board (MSRB)

The MSRB regulates all matters related to the underwriting and trading of state and municipal securities. While they have the authority to write the MSRB rules and regulations, they have no enforcement powers. LO 10.f D

Financial Industry Regulatory Authority (FINRA) staff must submit new rules and await approval from which of the following regulatory bodies prior to becoming effective? A) New York Stock Exchange (NYSE) B) Chicago Board Options Exchange (CBOE) C) Securities and Exchange Commission (SEC) D) Municipal Securities Rule Board (MSRB)

The MSRB, NYSE, and CBOE are deemed self-regulatory organizations (SROs) and are on an equal footing with FINRA. All SROs must first petition the SEC for approval of a new rule. LO 10.e C

How often is net asset value (NAV) for a mutual fund determined? A) At least twice per day B) Constantly throughout the business day C) At least once per business day D) At least once per day

The NAV of a fund share is the amount the investor receives upon redemption. It must be calculated at least once per business day, and one of those calculations must occur after the close of trading on the U.S. exchanges. A typical fund calculates its NAV at the end of each business day (4:00 pm ET). LO 4.d C

Which of the following entities considers appeals of decisions made in department of enforcement (DOE) actions? A) Municipal Securities Regulatory Board (MSRB) B) The National Adjudicatory Council (NAC) C) Federal Bureau of Investigation (FBI) D) Federal Reserve Board (FRB)

The National Adjudicatory Council (NAC) hears appeals from DOE decisions. An NAC ruling may be appealed to the SEC, then to the appellate courts. LO 10.e B

Regarding the Regulation T requirement, which of the following is true? A) It is currently 25% but can be changed at any time by the FRB. B) It is currently 50% and must remain unchanged unless mandated by Congress. C) It is currently 50% but can be changed at any time by the Federal Reserve Board (FRB). D) It is currently 30% and must remain unchanged unless mandated by Congress.

The Regulation T initial margin requirement is currently 50%. While it has been so for many decades, it can be changed by the Federal Reserve Board anytime it deems appropriate to do so. LO 6.g C

Your customer retired two years ago at age 68. He recently took a job with a golf course cleaning carts. He would like to contribute to a retirement plan to accumulate additional money with the view to leave something to his grandchildren. You would most likely advise him to open A) a traditional IRA. B) a mutual fund. C) a Roth IRA. D) an annuity.

The Roth IRA would require after-tax (nondeductible) contributions but would allow earnings to accumulate tax deferred as in any retirement plan. Roth IRA distributions need not begin at age 72, and if holding period requirements are satisfied, all distributions are tax free. LO 6.e C

A well-known average that tracks the performance of smaller U.S. companies is which of the following? A) Russell 2000 Index B) Standard & Poor's 500 Index C) MSCI EAFE Index D) Dow Jones Industrial Average

The Russell 2000 Index tracks approximately 2,000 small-company stocks and serves as the benchmark for smaller companies in the overall market. LO 7.e A

The Windmill Growth Fund is composed of many stocks from a variety of large companies. It has a stated objective of capital appreciation from holding the stock of the large companies. If you wanted to compare the performance of the fund to the market, which of these indices would be the best? A) Wilshire 5000 B) S&P 400 C) S&P 500 D) EAFE Index

The S&P 500 is the standard benchmark for large cap stocks. The S&P 400 is for midcap stocks. The Wilshire 5000 is a broad-based-U.S. equity index that includes large, mid, and small cap stocks. The EAFE is an index for international equities. LO 7.e C

What is the maximum number of nonaccredited investors allowed in a Regulation D exempt transaction under Rule 506(b)? A) 35 B) 25 C) 50 D) None

The SEC does not require registration of an offering under Regulation D as long as there are no more than 35 nonaccredited investors. There is no limit to the number of accredited investors that may invest in the private placement. LO 1.c A

The ATOP Company is planning to offer shares of both common and preferred stock to the investing public in order to raise operating capital intended to be used for expansion. Which of the following laws enacted by Congress would be the most relevant when issuing these equity securities to the public? A) The Trust Indenture Act of 1939 B) The Securities Investors Protection Act of 1970 C) The Securities Act of 1933 D) The Investment Company Act of 1940

The Securities Act of 1933, is also known as the Paper Act, Prospectus Act, or New Issues Act. This federal law requires that issuers who want to raise capital by making a public offering of securities to the public, provide full and fair disclosure of all material facts about the company and the securities being offered. LO 1.a C

The U4 form requires registered representatives to disclose all of the following except A) 10-year residency history. B) any aliases. C) 10-year employment history. D) information on any criminal charges, even if found innocent.

The U- 4 form only requires a five-year residency history. LO 12.a A

According to the U.S. Commerce Department, the economy is in a recession when a decline in real output of goods and services lasts A) 6 months or more. B) 18 months or more. C) 9 months or more. D) beyond 12 months.

The U.S. Commerce Department defines a recession as a decline in real output of goods and services for six months or more. LO 9.a A

Gerald is 78 years old. He approaches you to help him invest $150,000. He would like the money to generate some income to help pay for his new hobby: massive, multi-player, online role-playing games. He is rather risk averse outside the digital world and wants the principal to remain largely intact when he leaves it to his heirs. Which of the following would you recommend? A) Jim's growth fund B) Benji U.S. Treasury income fund C) Junker's aviation fund D) Windmill balanced fund

The U.S. Treasury fund is most aligned with Gerald's objectives of income with capital preservation. The other funds have varying degrees of exposure to stocks, which are not in line with the Gerald's stated objectives for the money. LO 7.d B

The Uniform Practice Code (UPC) establishes uniform trade practices pertaining to all of the following except A) communications with the public. B) good-delivery procedures. C) settlement and ex-dates. D) don't know (DK) procedures.

The UPC established uniform trade practices and other guidelines for broker-dealers to follow when they do business with other member firms, including transaction settlement, good delivery, ex-dates, trade confirmations, and DK procedures. LO 10.e A

The law that provides the legal framework for state registration of securities is A) the Trust Indenture Act of 1939. B) the Uniform Securities Act. C) the Securities Act of 1933. D) the Securities Exchange Act of 1934.

The Uniform Securities Act provides a legal framework for the state registration of securities, as well as the registration requirements applicable to broker-dealers, investment advisers, investment adviser representatives, and registered representatives. LO 10.d B

Four of the best-known indices and averages are listed as follows. How do they rank from most to fewest issues in the index? I. Dow Jones Industrial Average II. NYSE Composite Index III. Standard & Poor's 500 IV. Wilshire 5000 A) IV, II, III, I B) I, IV, III, II C) II, III, I, IV D) III, II, IV, I

The Wilshire actually had about 3,800 stocks, but still the most on this list. The NYSE composite is around 1,900. The S&P 500 is actually about 500, and the Dow Jones industrials in 30. LO 7.e A

One of your clients wants to set aside some money for her niece, who just turned 30, but the client has some reservations. The client does not wish her niece's numerous creditors to have access to the money until after the client dies, but she wants her niece to have easy access to the money at that time. You recommend that she open A) a custodial account. B) a TOD registration on an account in her name. C) a TIC account. D) a joint tenants with rights of survivorship account.

The acronym TOD stands for transfer on death. It is used to facilitate the transfer of assets in an account upon the death of the account holder (your customer, in this case) without the need for probate. While the owner is alive, the account remains her property. The niece is too old for a custodial account. She should not have a joint account because of creditor and control issues. LO 6.a B

According to the Uniform Securities Act, who is charged with enforcing state securities laws and regulations? A) NASAA B) The administrator C) The SEC D) FINRA

The administrator handles state securities laws. NASAA is an association of state administrators. The SEC and FINRA are both national level organizations. LO 10.d B

A bond with a 3% stated yield and a $1,000 par value would pay how much in annual interest? A) $3 B) $300 C) Not determinable D) $30

The amount of interest payable annually as the stated, nominal, or coupon yield is calculated as follows: rate 3% × par value ($1,000) = $30. LO 3.a D

New Haven Farms is a producer of specialty foods. They recently received a notice that the Wetlands Maintenance and Drainages Act (The WMD Act) passed indicates that a significant portion of their current land used in food production falls under the act's protection and may no longer be used for agriculture. This is an example of A) business risk. B) political risk. C) legislative risk. D) regulatory risk.

The best answer is legislative risk, as this was caused by a change in the law. Regulatory risk is a change in the application of existing rules, while political risk is normally associated with a change in leadership. Business risk would be a bad business plan, not a working business plan damaged by changing laws. LO 7.a C

The rate at which banks lend to broker-dealers for the purpose of lending money for margin loans is typically A) notably above (several percentage points) other short-term lending rates. B) slightly above (a percentage point or so) other short-term lending rates. C) notably below (several percentage points) other short-term lending rates. D) slightly below (a percentage point or so) other short-term lending rates.

The broker call loan rate is the rate at which banks lend to broker-dealers for the purpose of lending money for margin loans. This rate is usually slightly above, by a percentage point or so, other short-term lending rates. LO 9.i B

Five years ago Thompson, an investor, ran across a board game that he enjoyed and believed the game would become very popular. He purchased 1,000 shares of the corporation that publishes the game. Unfortunately, the game was too complex for most casual game players and sales never amounted to much. Over the five years, the stock of the publisher has remained steady, but has not increased in value. This is an example of A) timing risk. B) business risk. C) social risk. D) regulatory risk.

The failure of the game is an example of business risk. As a business venture the publisher did not do well. Timing and regulation had nothing to do with the failure. Social risk assumes a change in societal attitudes, in this case, there was no change, and the publisher and Thompson just read the market wrong. LO 7.a B

The rate that commercial money center banks charge each other for overnight loans is A) the broker call loan rate. B) the discount rate. C) the federal funds rate. D) the prime rate.

The federal funds rate is the rate commercial money center banks charge each other for overnight loans of $1 million or more. LO 9.i C

It is generally agreed upon that the most volatile interest rate in the U.S. economy is A) the discount rate. B) the federal funds rate. C) the prime rate. D) the call money rate.

The federal funds rate is the rate the commercial-money-center banks charge each other for overnight loans of $1 million or more. It is considered a barometer of the direction of short-term interest rates, which fluctuate constantly. Therefore, the federal funds rate can be considered the most volatile rate in the economy. LO 9.i B

All of the following regarding the firm element of a broker-dealer's continuing education (CE) requirement are true except A) it is applicable to both registered representatives and registered principals. B) it is prepared by Financial Industry Regulatory Authority (FINRA) for the member firm to administer. C) it must take into account the scope of the member firm's business. D) it comprises training for those who have direct contact with the public.

The firm element of a broker-dealer firm's continuing education (CE) requirement is prepared by the member firm. It comprises training for those personnel who have direct contact with the public, and it must be completed by all registered persons. Among other things, it must take into account the scope of the firm's business. LO 12.d B

A mutual fund company may offer noncash compensation to associates of broker-dealer firms in the form of attendance at a meeting or convention, provided that A) attendance is conditional upon agreement to a predetermined sales target. B) expenses of spouses or other guests of attendees are also met. C) a record of compensation and meeting details is kept by the attendee member's firm. D) the meeting is held for the purpose of entertainment only and not for business purpose.

The firms whose associates attend the meeting must keep records of all noncash compensation and details of what went on at the meeting. Noncash compensation of this type will inevitably be at least indirectly business-related, but must not be conditional upon agreeing to meet some sales goal. Side trips and expenses of guests must be met by those in attendance, not the host of the meeting. LO 12.f C

A customer purchased 100 shares of GHI common stock for $20 per share. After a year, they sell the shares for $23 per share. Over the past two years, GHI has paid a $0.25 quarterly dividend. What is the total return? A) 15% B) 10% C) 25% D) 20%

The formula for calculating total return is (income + gains or - losses) / cost basis. For this question ($1 + $3) / 20 = 4 / 20 = 0.20 (20%). Note that the position was held for only one year. LO 8.c D

A customer purchased 100 shares of BOB common stock for a total of $3,000. After one year, they sell the position for $3,200. BOB paid $1 per share in dividends over the year. What is the total return? A) 25% B) 15% C) 10% D) 20%

The formula for calculating total return is (income + gains or - losses) / cost basis. For this question ($100 + $200) / 3000 = 300 / 3000 = 0.10 (10%). LO 8.c C

A customer purchased 100 shares of MNO common stock two years ago for $20 per share. They now sell the shares for $23 per share. Over the last two years MNO has paid a $0.25 quarterly dividend. What is the total return? A) 25% B) 20% C) 15% D) 10%

The formula for calculating total return is (income + gains or - losses) / cost basis. For this question ($2 + $3) / 20 = 5 / 20 = 0.25 (25%). Note that the question does not ask for annualized return. LO 8.c A

A customer sells short 100 shares of ABC common stock at $35 per share. ABC stock increases in value, and the customer covers the short for $45 per share. What is the result of this set of transactions? A) $10 per share gain B) $10 per share loss C) $5 per share loss D) $5 per share gain

The formula for capital gains is sales proceeds - cost basis = gain (or loss). For this problem $35 - $45 = $-10 (a loss). Note that this is a short sale, so the sale was at $35 and the purchase at $45. LO 8.b B

Your customer purchases 100 shares of JIM, Inc., common stock at $75 per share on January 17, 2020. The customer sells the shares for $80 per share on February 2, 2021. The result of the sale is A) a $500 long-term capital loss. B) a $500 short-term capital gain. C) a $1,000 long-term capital gain. D) a $500 long-term capital gain.

The formula for capital gains is sales proceeds - cost basis = gain (or loss). For this problem $8,000 - $7500 = $500 gain. The position was held for more than one year so it is a long-term gain. LO 8.e D

SmallTown 8% bonds are currently trading at 100. What is the current yield? A) 6% B) 8% C) 4% D) 16 %

The formula for current yield is annual income ÷ current market value. In this example, the bond is trading at par (100), so the current yield is equal to the nominal yield (8%). LO 8.a B

XYZ Corp pays a quarterly dividend of $1. The common stock is currently valued at $160 per share. What is XYZ common stock's dividend yield? A) 2.5% B) 1.0% C) 7.5% D) 5.0%

The formula for dividend yield (or current yield) is annual income ÷ current market value. In this example you need to multiply the quarterly dividend by four to find the annual number: 4 ÷ 160 = 0.025 (2.5%). LO 8.a A

Your customer is in the 30% federal tax bracket. They consider purchasing a 7% corporate bond. Their after-tax yield would be A) 7%. B) 2.1%. C) 10%. D) 4.9%.

The formula for the calculation is 7% (corporate rate) × (100% — 30% (tax bracket)). 7 × (1 - 0.3) = = 7 × 0.7 = 4.9% LO 3.d D

One characteristic of an open-end investment company that distinguishes it from a closed-end one is that A) there is a continuous public offering. B) it may be either diversified or nondiversified. C) there are a wide variety of objectives available for investors to select from. D) it may avoid taxation by distributing all of its net investment income to shareholders.

The key difference between open-end investment companies and closed-end investment companies is the fact that new shares are continuously being offered for open-end companies. In the case of the closed-end, once the IPO is over, the only way to acquire shares is in the secondary market. Both types of funds may operate as regulated investment companies and avoid taxation, both may choose to be diversified or not, and both offer a wide variety of investment objectives. LO 4.a A

At what minimum age could normal distributions be made from an individual retirement account (IRA) without a penalty? A) 59½ years old B) 65 years old C) 62 years old D) 70½ years old

The minimum age to withdraw funds from a tax-qualified plan without penalty is 59½. Before that age, withdrawals are subject to a 10% penalty on growth withdrawn, unless an exclusion applies. LO 6.e A

Which of the following agencies collects and disseminates intelligence on financial transactions? A) The Office of the Comptroller of the Currency B) The Financial Crimes Enforcement Network C) The Securities Investors Protection Corporation D) The Internal Revenue Service

The mission of the Financial Crimes Enforcement Network (FinCEN) is to safeguard the financial system from illicit use; to combat money laundering; and to promote national security through the collection, analysis, and dissemination of financial intelligence. Most financial services companies are required to report suspicious or unusual activity to FinCEN. LO 10.b B

The mission of the Financial Crimes Enforcement Network is to A) analyze foreign trade. B) investigate and prosecute fraud. C) enforce anti-money laundering laws. D) safeguard the financial system from illicit use.

The mission of the Financial Crimes Enforcement Network (FinCEN) is to safeguard the financial system from illicit use; to combat money laundering; and to promote national security through the collection, analysis, and dissemination of financial intelligence. Most financial services companies are required to report suspicious or unusual activity to FinCEN. Suspicious activity reports and cash transaction reports go to FinCEN. LO 10.b D

Your clients, spouses Jared and Joaquim, are both 33 years old. They are concerned about saving money for a down payment on a home. Currently they have $120,000 set aside for that purpose. They plan to start a family in about a year and would like to buy the home in six months. The amount they have saved is more than 20% of the value of the homes in the area they want to move. They look to you for a place to save the money until they need it. Which of the following is the most suitable recommendation for the couple? A) A 26-week T-bill B) The Oldtime Life Insurance Company Variable Annuity C) The Benji Money Market fund D) The Windmill Growth Fund

The money market and the T-Bill are safe investments for this short time frame. However, the money market provides the couple with flexibility to withdraw early or leave it in longer should their needs change. Variable annuities are a retirement savings vehicle. The growth fund is subject to market downturns. LO 7.d C

Your client, Dana McRae, has no investment experience. She just retired and won't be in a high tax bracket. She also is concerned about volatility of her investments. She is more concerned about preserving her principal than getting a high rate of return. Which of the follow would you recommend? A) A money market mutual fund investing in short-term corporate debt B) A newly issued U.S. Treasury bond C) A long-term U.S. government bond mutual fund D) A money market mutual fund investing in short-term debt from city and county governments in her state

The money market funds have a stable $1 value, but she doesn't need the municipal money market because she isn't in a high tax bracket. The other two options will have significant volatility. LO 7.d A

The most commonly referenced indicator of economic activity is A) GDP. B) CPI. C) GNP. D) Dow.

The most commonly referenced economic indicator that measures overall economic activity is gross domestic product. Gross national product is not covered with anywhere near the frequency of GDP. CPI is a measure of inflation, not activity. Dow is a company or a set of indices, but not a measure of economic activity. LO 9.b A

The maximum gain on a long put is A) strike price + premium. B) the strike price. C) strike price - premium. D) the premium.

The most money that can be made on a long put is when the underlying stock falls to zero. The put can then be exercised. The holder of the long put sells the stock for the strike price, making the difference between the strike price and zero (remember, the stock is worthless). The holder still needs to account for the cost he paid for the option, so the formula is strike price - premium. Note that this is the same as the breakeven price. LO 5.a C

Which of the following documents must be provided to the customer prior to approval of an options account? A) Options Disclosure Document B) Official statement C) Statement of additional information D) Prospectus

The option disclosure document (typically referred to as the ODD) must be provided prior to account approval. LO 5.c A

Which of these places the phases of the business cycle in the correct order? A) Contraction, trough, expansion, and peak B) Expansion, contraction, trough, and recovery C) Peak, contraction, recovery, and prosperity D) Trough, recovery, contraction, and prosperity

The order should be peak (or prosperity), contraction, trough, recovery or expansion. For the test, the business cycle may begin with any of these four phases. Officially, business cycles begin with a recovery, but this question asks you to put the phases in the correct order without regard for where in the cycle we start. LO 9.a A

The penalties for trading on insider information include civil penalties up to treble damages. up to $1 million for registered representatives or broker-dealers. criminal penalties up to $25 million for broker-dealers. jail time up to 30 years. A) I and IV B) I and II C) I, II, and III D) III and IV

The penalties are stiff. Regulators really don't want people trading on inside information. However, the maximum prison sentence is 20 years, not 30. LO 11.d C

An investor requests a preliminary prospectus for a new issue. Regarding the document which of the following is true? A) The final price for the securities is published within it. B) Receipt of it is a commitment that the underwriters will sell securities to the recipient. C) It can be deemed an offer to sell securities to the public. D) It is made available between the registration date and the effective date.

The preliminary prospectus (red herring) is a prospecting tool used to gauge indications of interest. It is made available to those who request it between the registration date and the effective date (cooling-off period). Receiving it is not a commitment to purchase shares and making it available is not a commitment to sell shares to the recipient. No final price would be found on a preliminary prospectus. LO 1.b D

In which circumstances may a FINRA member firm conduct securities business with a nonmember broker-dealer? A) If the nonmember broker-dealer is a foreign firm and properly registered in their home jurisdiction B) If the broker-dealer is a Canadian firm without regard to registration C) If the broker-dealer is temporarily suspended and any transaction is related to ongoing business D) Under no circumstances

The properly registered foreign member exception is one of the few exceptions to the rule. A suspended member is also treated as a nonmember. LO 10.e A

A mutual fund has been in existence for 15 years. The prospectus must disclose the fund's performance A) over the last 1, 5, and 10 years. B) broken out as an average over the last 10 years. C) for each year over the last 10 years. D) over the last 1, 5, 10, 15, 20, and 25 years.

The prospectus of a mutual fund must show the fund's performance over the last 10 years or the life of the fund, whichever is shorter. The data must be shown as the last year's performance, the performance over the last 5 years, and the performance over the last 10 years. With this fund, the 15-year performance need not be shown. LO 4.e A

Records of original entry must be recorded no later than the next business day and must be kept readily available for A) six years. B) three years. C) two years. D) four years.

The records must be maintained for a period of six years, but must be readily available for two years. LO 10.g C

The minimum initial requirement when purchasing 100 shares at $30 in a new account would be A) $1,500. B) $375. C) $750. D) $2,000.

The requirement is normally 50% but not less than $2,000 unless the purchase price is less than $2,000. Then only 100% of the purchase price would be required. LO 6.g D

The variable return within a variable annuity is based on the performance of which of the following? A) General account B) S&P 500 C) Separate account D) Stock market

The return of a variable annuity is based on the performance of the separate account. The performance of the separate account is based on the investments held within the separate account. LO 4.f C

In order for a gain to be taxed as a long-term capital gain, the position must be held for how long? A) One year or more B) More than one year C) More than two years D) One year

The rule is "more than one year." If the test has calendar dates in the question, it will be clearly more than one year. Also, anything that protects the position (e.g., a long put) stops the clock for determining the one year. LO 8.e B

Under rules found in the Investment Advisers Act of 1940, if a political contribution to certain elected officials or candidates is made then an adviser may A) not provide advisory services to any government they represent for a fee until the next election. B) have to suspend the providing of advisory services to any government they represent until it is determined if the election campaign was successful or not. C) not provide advisory services to any government they represent for a fee for two years. D) not provide advisory services to any government they represent for a fee for three years.

The rule makes it unlawful for an adviser to receive compensation (i.e., fees) when providing advisory services to a government entity for a two-year period after the adviser makes a political contribution to a public official of a government entity. This includes candidates for office. The outcome of an election has no relevance within the rule. LO 12.f C

A member firm believes that financial exploitation of a senior customer is being attempted. While not required to, but allowed to, they put a temporary hold on disbursements of cash and securities from the account. The temporary hold should be A) no longer than 3 business days unless a state regulator or agency of jurisdiction extends it longer. B) for as long as is required for a complete and thorough investigation to be concluded. C) no longer than 15 business days unless a state regulator or agency of jurisdiction extends it longer. D) no longer than 15 business days with no extensions possible.

The rule specifies that the temporary hold on disbursements if one is placed should be for no longer than 15 business days. A state regulator or agency of jurisdiction can terminate it sooner or extend it longer if they deem it warranted. LO 11.f C

In order to lower the rate of inflation, the Federal Reserve might take which of the following steps? A) Decrease the reserve requirement B) Increase the prime rate C) Sell federal debt D) Purchase federal debt

The sale of Treasury debt by the Federal Reserve Board (FRB) decreases the liquid cash in the economy, slowing spending and decreasing demand. Purchasing debt would have the opposite effect. Decreasing the reserve requirement would also increase money supply and spur demand. The FRB does not control the prime rate. LO 9.h C

A mutual fund can offer all of the following to investors except A) physical custody of the fund's portfolio cash and securities. B) acting as custodian for retirement accounts. C) the ability to do transfers by telephone or online. D) check-writing privileges for redemptions.

The services mutual funds offer may include retirement account custodianship, investment plans, check-writing privileges, transfers by telephone or online, withdrawal plans, and a number of other services and privileges. However the Act of 1940 requires that each investment company place portfolio cash and securities with a custodian for safekeeping. LO 4.b A

Underwriters who are assisting an issuer in bringing securities to the investing public can do which of the following between the time the registration was filed with the Securities and Exchange Commission (SEC) and the effective date? A) Mail sales literature to those who have expressed an interest in purchasing the securities. B) Distribute a preliminary prospectus to the investing public. C) Make a binding offer to sell the securities. D) Solicit orders from investors to purchase the securities.

The time between the registration filing date with the SEC and the effective date is known as the cooling-off period. During this time, a preliminary prospectus may be distributed to gauge investor interest but no offers to sell the securities can be made and no orders to purchase the securities can be taken. While a preliminary prospectus and tombstone ad can be used, sales and advertising literature specific to the securities cannot be. LO 1.b B

The transfer agent for a corporation is responsible for each of the following except A) ensuring that its securities are issued in the correct owner's name. B) maintaining records of ownership. C) canceling old and issuing new certificates. D) acting as an intermediary between the buy and sell sides of a transaction.

The transfer agent (often a bank) for a corporation is responsible for ensuring that its securities are issued in the correct owner's name, canceling old and issuing new certificates, maintaining records of ownership, and handling problems relating to lost, stolen, or destroyed certificates. Acting as an intermediary in a trade is the function of the clearing corporation. LO 1.e D

Many investors like to put a transfer on death (TOD) designation on their brokerage accounts. Which of these are benefits of doing so? I. The TOD designation avoids estate taxes. II. The TOD designation avoids probate. III. The account holder is relieved of decision making in the account. IV. There is flexibility to change beneficiaries as conditions dictate. A) II and IV B) I and III C) II and III D) I and IV

The transfer on death (TOD) designation allows the account holder to name a specific beneficiary (or beneficiaries) to receive the account's assets upon death. Those named persons may be changed whenever the account holder wishes. Although this bypasses probate, it does not avoid estate taxes. TOD has nothing to do with giving investment discretion. LO 6.a A

An investor is interested in supplementing retirement income, is comfortable with market risk, and is concerned about the effects of inflation on their retirement cash flow. Which of the following securities is suitable for this investor? A) A variable annuity B) A Treasury bond fund C) An international bond fund D) A fixed annuity

The variable annuity is the only option available in this question that meets all three criteria. Bonds and fixed annuities generate a fixed income and do not have the ability to grow with inflation. LO 4.g A

Modulux, Inc., a NYSE listed manufacturer, is offering 5 million shares to the public, which will raise capital to build a new plant. The new technology and design should allow Modulux to increase market share significantly in the modular home business. This offer is A) an IPO. B) an APO. C) a secondary offering. D) a venture offering.

There are two important points to consider (1) This is an offering of stock to raise money for the issuer, which is (2) a primary transaction. The company's stock is actively trading in the secondary markets so this must be additional shares. This is an additional public offering (APO). LO 1.a B

What is the penalty for not taking the required minimum distribution (RMD) for the year? A) 10% of the amount that should have been taken B) 50% of the amount short of what should have been taken C) 10% of the annual contribution limit D) 50% of the annual contribution limit

There is a 10% penalty for early withdrawal. The penalty for missing a RMD is 50% of the amount missed. LO 6.e B

A state government has outstanding debt that it issued to finance toll roads, sports facilities, and public housing projects. All of these issues are examples of A) Treasury bonds. B) convertible bonds. C) municipal revenue bonds. D) municipal general obligation (GO) bonds.

These are all examples of municipal revenue bonds, which are bonds issued to finance a project or facility with the bonds' debt service backed by the facility's revenue stream. The revenue might come from rents, tolls, or admission fees, among other sources. LO 3.d C

The following records must be maintained by a broker-dealer: partnership records, articles of incorporation, records of the board of directors, Form BD, and amendments to the form. How long must they be maintained by the firm? A) Four years from the creation of the record B) As long as the firm is in business C) Six years from the creation of the record D) Two years form the creation of the record

These records are considered to define the firm and must be kept as long as the firm is in business. LO 10.g B

Several registered representatives have agreed to purchase a certain security for their own accounts, then tout the security to their best customers. They hope that a number of large orders will be entered, driving up the price of the security, which they can then sell from their own accounts for a capital gain. This illegal practice is a form of A) insider trading. B) front running. C) pump and dump. D) supporting.

These salespersons are engaged in a pump and dump scheme. In this prohibited practice, the registered representatives buy shares for their own accounts and then tout the stock to customers in the hopes that the customers' buying will drive the stock price higher. The reps then sell their personal positions for an illicit profit at the customers' expense. LO 11.c C

Your customer has performed the following trades Bought 200 shares of ABC at $40 Bought 400 shares of ABC at 50 Sold 600 shares of ABC at 55 What is the result of these trades? A) A $6,000 loss B) A $6,000 gain C) A $5,000 loss D) A $5,000 gain

They bought 200 at 40 for $8,000; then 400 at 50 for $20,000; then sold 600 at 55 for $33,000. $33k - $28k = profit of $5,000. LO 8.b D

Your customer purchased 1,000 shares of SmallCo Stock at $10 a share. SmallCo pays no dividends. Exactly one year later, the customer sold the shares for $12 a share. They realized a A) $2,000 long-term capital gain. B) $2,000 long-term capital loss. C) $2,000 short-term capital gain. D) $2,000 short-term capital loss

They bought the shares for $10, and sold for $12, so a $2,000 gain. To be a long-term gain the position must be held for more than one year. LO 8.e C

Which of the following would not be an investment recommendation? A) In response to a customer's request about growth investments, a representative provides information on the Windmill Growth Fund. B) In response to a customer's request about the Windmill Growth Fund, a representative provides information about the fund. C) A representative contacts an investor and introduces the customer to the Windmill Growth Fund. D) During a presentation to a group, a representative uses a chart of the performance of the Windmill Growth Fund as an example of a superior growth investment.

This is simply fulfilling a customer's request for information on a security they are interested in; the firm has not presented or recommended the security. In all the other responses the representative introduced the security at some level. LO 7.c B

Joshua, a registered representative, calls a prospect at 7:00 pm only to be told that the prospect is in a different time zone, where it is 10:00 pm. Joshua has done which of these? A) Not violated the TCPA because it is still within the allowed time at his location B) Not a violation because the rule is no longer valid due to emerging mobile technology C) Not a violation because the area code was local to Joshua's location D) Violated the TCPA because it is after 9:00 pm the prospect's time

This is still a violation. The rule clearly states that the time requirements are based on the prospective customer's time. The existence of cell phones does not invalidate the law. LO 11.j D

Assets = liabilities + net worth is the A) basic balance sheet equation. B) price to book ratio. C) working capital formula. D) fully diluted earnings ratio.

This is the basic balance sheet equation: assets = liabilities + net worth. LO 9.d A

The strike price is A) the amount the buyer of a call must pay for the shares. B) the cost per share of the contract. C) the amount the writer of a call pays the seller of the call for the shares. D) the price that will be paid for the shares if the option is exercised.

This is the basic definition of strike price. The cost per share of the contract is the premium. The terms writer and seller are the same person in an option transaction. The buyer of a call may choose to exercise and buy the stock, or they may choose not to do so. They have the right (may) but not the obligation (must) to exercise the contract. LO 5.a D

When a broker-dealer borrows money to fund margin purchases, the rate for this loan is called A) the broker call loan rate. B) the reserve rate. C) the prime rate. D) the margin interest rate.

This is the broker call loan rate. The prime rate is the rate banks charge their best corporate customers for short-term loans. BDs charge margin customers the margin interest rate. There is no reserve rate in this context. LO 9.i A

A business entity that performs the function of receiving and delivering payments and securities on behalf of both parties to a securities transaction is called a A) depository. B) clearing agency. C) transfer agent. D) broker-dealer.

This is the function of a clearing agency. Although there are some broker-dealers that do act as clearing agents, being a broker-dealer does not always include providing the services of a clearing agent. The broker-dealer would need to meet all of the requirements of being a clearing agent. LO 1.e B

Both the individual and institutional investor are able to easily buy and sell securities to meet their objectives through A) biennial markets. B) outdoor malls. C) primary markets. D) secondary markets.

This is the primary purpose of the secondary markets. LO 1.d D

The Hoffman Equipment Company of Anaheim, California is preparing to sell shares to the public in an IPO. They plan to sell exclusively to residents of the state of California and use the proceeds to build a new showroom in Long Beach, California. This offer would need to be registered with A) the SEC. B) MSRB. C) no one; this is an exempt transaction. D) the administrator of the state.

This offer would be exempt from registration with the SEC because it meets the criteria under the Intrastate Offer Rule, but would still need to be registered with the state. MSRB makes rules for municipal securities. LO 10.d D

A customer sells shares of the ABC Growth Fund and invests the proceeds into the Windmill Income Fund. Both investments are in Class A shares. What are the tax consequences for these transactions? A) The customer will defer any gains and will not pay a new sales charge. B) The customer will realize any capital gains or losses and will pay a new sales charge. C) The customer will realize any capital gains or losses but will not pay a new sales charge. D) The customer will defer any gains and will pay a new sales charge.

This transaction will result in the customer realizing any capital gains or losses for tax purposes. The exchange (conversion) privilege would waive the new sales charge if this transaction was between funds within the same fund family (sponsor). In this question the funds are at different sponsors; any sales charges will apply to the new purchase. LO 4.b B

An investor is long a call option. Over time, the underlying security rises in value above the strike price of the call. It is likely that the call would A) have no intrinsic value. B) not be exercised. C) be exercised. D) decline in value.

Those who own call options want the value of the underlying stock to go above the strike price of the call. The current market value of the underlying going above the strike price makes the option contract in the money (it has intrinsic value). The investor would either exercise the option or sell it for a profit. LO 5.a C

Your customer asks to buy a bond that carries a very attractive yield. When checking the bond, you see that it has a B rating from the major credit rating agencies. When communicating such information to a customer, all of the following terms are commonly used in describing a B-rated bond except A) junk bond. B) noninvestment grade. C) lower grade. D) high-yield.

Though a B rating is certainly a lower investment-grade rating, that is not a typical term used in the industry. All of the other terms are terms normally associated with these bonds carrying a greater risk of default. LO 3.b C

Another term for a defined benefit plan is A) a traditional term plan. B) a pension plan. C) an annuity. D) a defined withdrawal plan.

Though an annuity acts much like a defined benefit plan, it does not require sponsorship by an employer. The other two responses are just made up. LO 6.f B

A customer purchased 500 shares of GMO, Inc., common stock at $40 per share. The customer held the position for two years before selling the position at $40 per share. During the time the customer held GMO stock, the company paid a steady quarterly dividend of $0.25. What was the customer's total return for this position? A) 0% B) 2.5% C) 5% D) 1.25%

Total return is calculated over the holding period, two years in this example. The company paid a $0.25 quarterly dividend for two years for a total income of $2 over the period. The cost of the stock and the sale price are the same; the customer neither received capital gains nor incurred any capital loss. To calculate the total return, you would add any gains or subtract losses from any income received, then divide the result by the purchase price. In this example [(0.25 × 8)+(40 - 40)]/40 = (2+0)/40 = .05 (5%). LO 8.c C

All of the following are records that are kept for the life of the firm except A) board meeting minutes. B) stock book. C) trade blotters. D) articles of incorporation.

Trade blotters are maintained for six years. Documents that are related to the creation of the firm and its basic organizational structure are normally kept for the life of the firm. LO 10.g C

Which of the following is true regarding a registered person who wishes to move her registration from one broker-dealer to another? A) A person who has been registered more than 25 years is grandfathered in and no filings need to be made. B) Only Form U5 need be filed, separating the registered person from the existing employer. C) In no circumstances can a registration be transferred from one firm to another. D) If the registered person has taken and passed the Series 24 General Securities Principal Exam, no filings need be made.

Transferring a registration from one member firm to another is not permitted. Should a person resign or be terminated, the member firm must file a Form U5 with the Central Registration Depository (CRD) within 30 days of the termination date. A Form U4 must then be filed by the new employer with all of the form's information requirements met. LO 12.a C

Which of these statements regarding Treasury bills is correct? A) They are usually issued at a slight premium to par value. B) They are issued with initial maturities of 3, 12, 24, and 50 weeks. C) They have the highest interest rate risk of all Treasury securities. D) Treasury bills are the only type of Treasury security issued without a stated interest rate.

Treasury bills are always issued at a discount, without a stated interest rate. Receiving par value back at maturity represents the interest income to the investor. Because of their short-term maturities, they have the lowest interest rate risk for Treasury securities, not the highest. T-Bills are issued in initial maturities of 4, 13, 26, and 52 weeks. LO 3.e D

Tony was fired for cause from Great Plains Investments. A U5 Form was promptly filed stating that the cause was rude treatment of coworkers creating a hostile work environment. Great Plains will keep the U5 record for a minimum of A) three years. B) two years. C) six years. D) indefinitely.

U5s have a three-year retention requirement. LO 10.g A

One difference between an UTMA account and an UGMA account is A) a UTMA account transfers at the age of majority, while a UGMA account can go until the beneficiary turns 30 year of age. B) a UTMA account has a wider set of allowed investments. C) UTMA assets are considered an irrevocable gift, while a UGMA account allows the custodian to reclaim the assets. D) a UGMA account is tax deferred, and a UTMA account is not.

UTMA accounts allow real estate holdings; a UGMA account does not. Neither account is tax deferred and both accounts are irrevocable. The UGMA transfers at the age of majority; the UTMA may transfer as late as age 25. LO 6.d B

When the Options Clearing Corporation (OCC) assigns exercise notices to a broker-dealer, the broker-dealer will in turn assign the exercise to customers with short positions using any of the following methods except A) using a random selection method. B) using any method that is considered fair and reasonable. C) on a LIFO basis. D) on a FIFO basis.

Under OCC rules, LIFO is not considered an acceptable method for assigning an exercise to a customer on the short side of an options trade. The others listed are all acceptable. LO 5.c C

Under the Investment Company Act of 1940, all of these are examples of management companies except A) a Windmill Income UIT. B) an S&P 500 Index Trust ETF. C) a Windmill Income Fund, an exchange-listed:closed-end fund. D) a growth fund option for a VA.

Unit investment trusts are investment companies, but not management companies under the act. Closed-end funds, ETFs, and separate accounts are all types of management companies. LO 4.a A

Banker's acceptances are A) intermediate-term debt instruments issued by the federal government to banks. B) short-term time drafts issued by banks to the federal government. C) short-term time drafts issued by banks to corporations. D) intermediate-term debt instruments issued by banks to corporations.

Used by corporations to finance international (foreign) trade, BAs are issued by banks to corporations. They are considered money market instruments because of their short-term maturities, generally no longer than 270 days (9 months). LO 3.f C

A variable annuity (VA) is an investment product designed to provide which of the following? A) Speculative gains B) A short-term savings vehicle C) A supplement to retirement D) A tax-free income source

VAs are designed to provide a supplement to retirement income. Most VAs have surrender charges that make them unsuitable for short term investments. These investments are tax deferred, not tax free. Subaccounts are investment company products and are not speculative in nature. LO 4.f C

First Amalgamated Bank of Buffalo, a large commercial bank, is a member of the Federal Reserve System. Should the bank need to increase its reserves, it could do which of these? I. Borrow from the FRB and pay the discount rate. II. Borrow from the FRB and pay the federal funds rate. III. Borrow from another member bank and pay the discount rate. IV. Borrow from another member bank and pay the federal funds rate. A) I and IV B) II and IV C) II and III D) I and III

When a bank needs to borrow money to increase its reserves, it can borrow from the Federal Reserve Bank or it can borrow from another member bank like itself. When borrowing from the FRB, the banks pay the discount rate. When borrowing from another member bank, the banks pay the federal funds rate. LO 9.i A

Seacoast Securities, a member firm, filled a customer's sell order for BigTech Computers common stock and did not charge a commission. The firm most likely acted in what capacity? A) Broker B) Dealer C) Underwriter D) Agent

When a broker-dealer acts as a principal (market maker, dealer), they are filling the trade through the firm's inventory. The firm profits on the difference between what the firm paid for the security and what the customer pays, which is called the spread (or mark-up). There is no commission when a firm acts in a principal capacity. LO 1.h B

A broker-dealer that concentrated its business efforts on proprietary trading would most likely be functioning as A) a market maker. B) an investment banker. C) an underwriter. D) an investment adviser.

When a broker-dealer buys and sells securities for its own account as the major portion of its business model (e.g., proprietary trading), it is functioning primarily as a market maker (i.e., making markets in those securities). Investment banking and underwriting both primarily involve assisting issuers with bringing new securities issues to public investors. Investment advisers sell advice, they do not trade securities. LO 1.h A

An investor purchased 100 shares of LMN stock in 2013 at a price of $40 per share. Soon after, LMN declared a 25% stock dividend. Three years after the shares were purchased, they were sold at $50. Which of the following statements are correct? I. The adjusted cost basis of the shares is $30. II. The adjusted cost basis of the shares is $32. III. There is a short-term capital gain on all the shares sold. IV. There is a long-term capital gain on all the shares sold. A) II and IV B) I and IV C) I and III D) II and III

When a company declares a stock dividend, the cost basis per share is always reduced. The customer will receive 25 new shares (100 shares × 0.25 = 25). The computation is the original total cost $4,000 (100 × $40) divided by the new number of shares 125 (100 + 25). Four-thousand dollars divided by 125 shares equals a new cost basis per share of $32. The holding period for capital gain or loss (short or long term) is always from the original purchase date. In this case, because the shares were sold three years later at 50, the gains are long term. LO 8.b A

A company offers to repurchase outstanding debt securities it has issued directly from its bondholders for cash in what would commonly be known as A) an acquisition. B) a buy back. C) a tender offer. D) a hostile takeover.

When a company offers to buy outstanding securities for cash or for cash plus other securities from its stockholders or bondholders this is known as a tender offer. In contrast, a buyback, sometimes referred to as a repurchase, is when a company buys its own outstanding securities in the open market rather than appealing directly to its investors. LO 2.i C

In a leasing partnership program, loans are taken to purchase equipment that is then leased to companies in return for the lease payments. This process A) eliminates any possibility of sheltering the income from the lease payments received with deductions or credits. B) enables the partners to take tax credits against the income received from the lease payments. C) allows for the loan interest and equipment depreciation to be taken as deductions that will shelter the income from the lease payments received. D) allows for the equipment to be depreciated, adding to the income realized by the partnership.

When a leasing program purchases equipment that it will lease to companies in return for the lease payments, the program can deduct over the life of the program any interest costs on the loans to purchase the equipment, as well as any depreciation on the equipment it owns and leases. These deductions shelter the income taken in from the lease payments. LO 5.f C

BigCo, Inc., issues a collateral trust bond. Which of the following statements about this bond is true? A) This is a secured bond backed by marketable securities owned by the issuer. B) This is a secured bond backed by rolling stock owned by the issuer. C) This is an unsecured bond backed by marketable securities owned by a third party. D) This is a secured bond backed by real estate owned by the issuer.

When issuing collateral trust bonds or certificates, an issuing corporation deposits marketable securities it owns into a trust in order to secure the loan. The securities it deposits can be securities in other corporations or those of partially or fully owned subsidiaries as long as the securities are marketable. The securities become the lender's (bondholder's) collateral. LO 3.c A

If large money center commercial banks begin to lower their prime rates, which of the following is most likely to occur? A) Smaller banks will need to offset the lower prime rate by increasing the broker call loan rate. B) Smaller banks will need to increase their lending rates for creditworthy corporate customers. C) Smaller banks will lower lending rates for creditworthy corporate customers as well. D) Smaller banks will follow by lowering the discount rate.

When large money center commercial bank lower the prime rate, the rate charged to their most creditworthy corporate customers, smaller banks will generally follow in order to stay competitive. The discount rate is set by the Federal Reserve Board (FRB) (not banks), and if the broker call loan rate banks charge is impacted, it would also be lowered (not increased). LO 9.i C

When making unsolicited cold calls to prospects, a registered representative must disclose all of the following to the individual called except A) the caller's name. B) the address at which the caller may be contacted. C) the name of the member broker-dealer firm. D) the address of any securities issuer mentioned during the call.

When making cold calls, the caller must disclose his name and the name of the member broker-dealer, the telephone number or address at which the caller may be contacted, and that the purpose of the call is to solicit the purchase of securities. When securities of any issuer are mentioned in such a call, there is no requirement to disclose the address of the issuer. LO 11.j D

When the Federal Reserve Board (FRB) wants to expand (loosen) the money supply, it will A) buy Treasury securities from banks in the open market. B) buy corporate securities from banks in the open market. C) sell corporate securities to banks in the open market. D) sell Treasury securities to banks in the open market.

When the FRB wants to expand (loosen) the money supply, it will buy Treasury securities from banks in the open market. The securities come out of the economy, and the money goes into the economy. LO 9.h A

A strong U.S. dollar leads to more A) U.S. exports and a balance of payments deficit. B) U.S. exports and a balance of payments surplus. C) U.S. imports and a balance of payments surplus. D) U.S. imports and a balance of payments deficit.

When the dollar is strong, it is more affordable for U.S. consumers to buy more foreign goods, so U.S. imports increase. As more imported goods flow in, more money flows out—deficit. LO 9.f D

When investing in overseas markets in foreign securities, investors should be aware of and understand A) reinvestment risk. B) currency risk. C) business risk. D) market risk.

Whenever investing in securities issued in non-U.S. markets, investors need to be sensitive to the different risks that might apply to foreign investments. Of those listed here, currency risk should be of concern. Currency risk is the possibility that an investment denominated in a foreign currency could decline for U.S. investors if the value of that currency declines in its exchange rate with the U.S. dollar. LO 7.a B

A final prospectus must include certain information. Which of the following is not required to be included? A) A history of the business including any risks to purchasers that might be unique to that business or industry B) The underwriting contract and a list of all underwriters named in the contract C) The intended use of the proceeds for such corporate actions including expansion, mergers, or acquisitions D) A statement by the SEC that the offering is neither approved or disapproved

While a description of the underwriting is required (this would include a list of the underwriters provided in the preliminary prospectus) the actual underwriting contract is not required in a final prospectus. LO 1.b B

Which of the following regarding income is true? A) Salary or bonuses are earned income; interest and dividends are investment income. B) Salary, bonuses, interest, and dividends are all investment income. C) Salary or bonuses are portfolio income; interest and dividends are investment income. D) Salary, bonuses, interest, and dividends are all portfolio income.

While someone's salary or bonus would be earned income, investment income is that which is earned from one's investments. Sometimes called portfolio income, it would include dividends, interest, and capital gains derived from the sale of securities. LO 8.d A

While the Municipal Securities Rule Board (MSRB) writes the rules and regulations regarding underwriting and trading for municipal securities, it does not enforce those rules. Who does? A) Federal Reserve Bank (FRB) B) New York Stock Exchange (NYSE) C) Options Clearing Corporation (OCC) D) Financial Industry Regulatory Authority (FINRA)

While the MSRB has rulemaking authority, it has no authority to enforce those rules. The enforcement of the MSRB rules on broker-dealers has been primarily delegated to FINRA. Bank regulatory agencies, such as the Federal Reserve Board (FRB), enforce the MSRB rules on banks who act as municipal dealers. LO 10.f D

An investor has been putting aside funds for retirement in a nonqualified variable annuity for over five years. She is now age 66 and takes a lump-sum distribution. How are the earnings taxed? A) As tax free B) As long-term capital gain, except for any portion contributed in the previous 12 months C) As long-term gain D) As ordinary income

With a nonqualified annuity, all distributions more than the cost basis will be taxed as ordinary income. LO 8.d D

XYZ Technology Corporation common stock currently trades at $60 per share and pays a $0.30 dividend. What is XYZ's current dividend yield? A) 1.5% B) 2% C) 1% D) 0.5%

XYZ pays a $0.30 quarterly dividend. Current (dividend) yield is an annual figure. To calculate the current dividend yield you would multiply the quarterly dividend by four and then divide the result by the current market value. (0.30 × 4)/30 = 1.2/60 = .02 (2%) LO 8.a B

Your customer, Ivan, owns a diversified portfolio of large cap stocks. He would like to find a way to hedge the market risk in his portfolio. Which of these actions might you recommend to accomplish his goal? A) Tell him to do nothing because there is no way to mitigate systematic risk. B) Sell all his stocks and wait for a better time. C) Buy S&P 500 index puts to hedge market risk. D) Diversify his portfolio further.

Your customer is worried about market risk, a systematic risk. Further diversifying his portfolio will not help. Selling everything is likely not helping, and means he may miss any upward moves while he is waiting it out. Derivatives like options (index puts in this case) may be used to offset market risk. LO 7.b C

The current quote for PQR Inc. common stock is bid 42, ask 42.10, 4 × 5. A customer who places an order to buy 200 shares at the market will likely pay how much for the purchase? A) $8,420 B) This cannot be determined based on this quote C) $4,210 D) $8,400

Your customer will likely buy at the ask (42.10) or very close to it. The ask size is 500 shares, so the market can absorb the entire order (42.10 × 200 = $8,420). Customers who are selling receive the bid. Those who are buying pay the ask. The size shows bid × ask: 400 shares may be sold at 42 and 500 shares are available to buy at 42.10. LO 1.f A

The current quote for All American Motors Corp. stock is bid 52-ask 52.05 5 × 2. Your customer places an order to sell 300 shares. How much will he likely receive, before commission? A) $15,615 B) Cannot be determined because only 200 shares are available on this side of the quote C) $5,200 D) $15,600

Your customer will sell at the bid (52), or likely very close to it. The bid size is 500 shares, so the market can absorb the entire order; 52 × 300 = $15,600. Customers who are selling receive the bid. Those who are buying pay the ask. The size shows bid × ask; 500 shares may be sold at 52 and 200 shares are available to buy at 52.05. LO 1.f D


Conjuntos de estudio relacionados

Exercise 10: The Appendicular Skeleton

View Set

Words Occurring More than 50 Times in the New Testament (from Kubo)

View Set

Consumer Behavior Learnsmart Ch. 11

View Set